You are on page 1of 91

Total Marks : 200

Test-30(Subject)
( INSTA Prelims Test Series 2021 )

1. Participatory Guarantee System for India is a scheme for

A. Mid-Day Meals
B. Pharma Sector
C. Migrant workers
D. Organic Foods

Correct Answer : D

Answer Justification :

Agriculture Ministry’s Participatory Guarantee Scheme (PGS)

What is the PGS, and how does it work?

PGS is a process of certifying organic products, which ensures that their production takes
place in accordance with laid-down quality standards. The certification is in the form of a
documented logo or a statement. Hence, option (d) is correct.

According to ‘Participatory Guarantee System for India [PGS-India]’, an ‘Operational Manual


for Domestic Organic Certification’ published in 2015 by the National Centre of Organic
Farming, Ghaziabad, under the Ministry of Agriculture’s Department of Agriculture and Co-
operation, PGS is a “quality assurance initiative that is locally relevant, emphasize[s] the
participation of stakeholders, including producers and consumers, and (which) operate[s]
outside the framework of third-party certification”.

According to a 2008 definition formulated by the International Federation of Organic


Agriculture Movements (IFOAM), the Bonn-based global umbrella organisation for the organic
agriculture movement, PGSs are “locally focused quality assurance systems” that “certify
producers based on active participation of stakeholders and are built on a foundation of trust,
social networks and knowledge exchange”.

PGS, according to this definition, is “a process in which people in similar situations (in this
case small holder producers) assess, inspect and verify the production practices of each other
and take decisions on organic certification”.

some limitations of PGS.

* PGS certification is only for farmers or communities that can organise and perform as a
group within a village or a cluster of continguous villages, and is applicable only to farm
activities such as crop production, processing, and livestock rearing, and off-farm processing
“by PGS farmers of their direct products”.

* Individual farmers or group of farmers smaller than five members are not covered under

www.insightsactivelearn.com 1
Total Marks : 200
Test-30(Subject)
( INSTA Prelims Test Series 2021 )

PGS. They either have to opt for third party certification or join the existing PGS local group.

* PGS ensures traceability until the product is in the custody of the PGS group, which makes
PGS ideal for local direct sales and direct trade between producers and consumers.

https://indianexpress.com/article/explained/explained-what-is-pgs-the-heart-of-the-organic-food
-production-industry-6021466/

2. Consider the following statements regarding Project ‘Pyrasol’


1. It was launched to transform urban organic waste into biochar and energy in smart cities.
2. It is collaboration between India and Israel.

Which of the statements given above is/are correct?


A. 1 only
B. 2 only
C. Both 1 and 2
D. Neither 1 nor 2

Correct Answer : A

Answer Justification :

Project ‘Pyrasol’:

Context:

The foundation stone of the integrated Solar Dryer and Pyrolysis pilot plant was recently laid
at Chennai.

The pilot is part of the Indo-German project ‘Pyrasol’, launched to transform urban
organic waste into biochar and energy in smart cities.

It was awarded to CSIR-CLRI by the Indo-German Science & Technology Centre. Hence,
statement 2 is incorrect.

The project will ultimately lead to technology development for the joint processing of
Fibrous Organic Waste (FOW) and Sewage Sludge (SS) of Indian smart cities into
hygienic and highly valuable biochar associated with energy recovery, carbon
sequestration and environmental improvement.

www.insightsactivelearn.com 2
Total Marks : 200
Test-30(Subject)
( INSTA Prelims Test Series 2021 )

Project Pyrasol:

The project focuses on managing and organising collection, treatment, and disposal systems of
urban wastes in Indian Smart Cities as well as in other urban centres with an integrated and
interactive approach.

About Indo-German Science & Technology Centre (IGSTC):

Established by India and Germany to facilitate Indo-German R&D networking with


emphasis on industry participation, applied research and technology development.

IGSTC through its flagship program ‘2+2 Projects’, catalyses innovation centric R&D
projects by synergising the strength of research and academic institutions and
public/private industries from India and Germany.

https://pib.gov.in/PressReleasePage.aspx?PRID=1714426

3. Consider the following statements regarding Departmentally-related standing committees (DRSCs)


1. Each DRSC has 31 members, 21 from Lok Sabha and 10 from Rajya Sabha.
2. Members are appointed for a maximum period of five years.

Which of the statements given above is/are correct?


A. 1 only
B. 2 only
C. Both 1 and 2
D. Neither 1 nor 2

Correct Answer : A

Answer Justification :

What are Parliamentary Committees?

A parliamentary committee is a “committee which is appointed or elected by the House or


nominated by the Speaker and which works under the direction of the Speaker and presents
its report to the House or to the Speaker and the Secretariat”.

Parliamentary Committees are of two kinds – Standing Committees and ad hoc


Committees. The former are elected or appointed every year or periodically and their
work goes on, more or less, on a continuous basis. The latter are appointed on an ad hoc
basis as need arises and they cease to exist as soon as they complete the task assigned
to them.

www.insightsactivelearn.com 3
Total Marks : 200
Test-30(Subject)
( INSTA Prelims Test Series 2021 )

Constitutional Provisions:

Parliamentary committees draw their authority from Article 105 (on privileges of Parliament
members) and Article 118 (on Parliament’s authority to make rules for regulating its
procedure and conduct of business).

Composition of Departmentally-related standing committees (DRSCs):

Until the 13th Lok Sabha, each DRSC comprised 45 members — 30 nominated from Lok
Sabha and 15 from the Rajya Sabha.

However, with their restructuring in July 2004, each DRSC now has 31 members —
21 from Lok Sabha and 10 from Rajya Sabha, to be nominated by Lok Sabha
Speaker and Rajya Sabha chairman, respectively.

They are appointed for a maximum period of one year and the committees are
reconstituted every year cutting across party lines. Hence, statement 2 is incorrect.

https://epaper.thehindu.com/Home/ShareArticle?OrgId=GUN8HEG71.1&imageview=0

4. Consider the following statements regarding Agriculture Infrastructure Fund


1. It is a short-term debt financing facility for investment in viable projects.
2. All loans under this financing facility will have interest subvention of 3%.

Which of the statements given above is/are correct?


A. 1 only
B. 2 only
C. Both 1 and 2
D. Neither 1 nor 2

Correct Answer : B

Answer Justification :

Agriculture Infrastructure Fund:

Context:

www.insightsactivelearn.com 4
Total Marks : 200
Test-30(Subject)
( INSTA Prelims Test Series 2021 )

Agriculture Infrastructure Fund has crossed the Rs. 8000 crore mark after receiving 8,665
applications worth Rs. 8,216 crores.

The largest share of the pie is contributed by Primary Agricultural Credit Societies
(PACS) (58%), Agri-entrepreneurs (24%) and individual farmers (13%).

About the Agriculture Infrastructure Fund:

It is a medium – long term debt financing facility for investment in viable


projects for post-harvest management infrastructure and community farming assets
through interest subvention and credit guarantee. Hence, statement 1 is incorrect.

The duration of the scheme is from FY2020 to FY2029 (10 years).

Under the scheme, Rs. 1 Lakh Crore will be provided by banks and financial institutions
as loans with interest subvention of 3% per annum and credit guarantee coverage under
CGTMSE for loans up to Rs. 2 Crores.

Eligible beneficiaries include:

Farmers, FPOs, PACS, Marketing Cooperative Societies, SHGs, Joint Liability Groups (JLG),
Multipurpose Cooperative Societies, Agri-entrepreneurs, Start-ups, and Central/State agency
or Local Body sponsored Public-Private Partnership Projects.

Interest subvention:

All loans under this financing facility will have interest subvention of 3% per annum up
to a limit of Rs. 2 crores. This subvention will be available for a maximum period of seven
years.

https://pib.gov.in/PressReleasePage.aspx?PRID=1656140

5. Exercise Varuna is a naval exercise between India and

A. Russia
B. UK
C. France
D. USA

www.insightsactivelearn.com 5
Total Marks : 200
Test-30(Subject)
( INSTA Prelims Test Series 2021 )

Correct Answer : C

Answer Justification :

Exercise Varuna 2021:

The Indian and French Navies have been conducting bilateral maritime exercises since
1993. Since 2001, these exercises have been called ‘VARUNA’.

2021 was the 19th edition of the exercise and it was held in the Arabian Sea.

Exercise Varuna - 2021

The 19th edition of the Indian and French Navy bilateral exercise ‘VARUNA-2021’ is
conducted in the Arabian Sea from 25th to 27th April 2021. Hence, option (c) is correct.

From the Indian Navy’s side, guided missile stealth destroyer INS Kolkata, guided missile
frigates INS Tarkash and INS Talwar, Fleet Support Ship INS Deepak, with Seaking 42B and
Chetak integral helicopters, a Kalvari class submarine and P8I Long Range Maritime Patrol
Aircraft, will participate in the exercise. The French Navy will be represented by the Aircraft
Carrier Charles-de-Gaulle with Rafale-M fighter, E2C Hawkeye aircrafts and helicopters
Caïman M and Dauphin embarked, Horizon-class Air defense destroyer Chevalier Paul,
Aquitaine-class multi-missions frigate FNS Provence with a Caïman M helicopter embkared
and Command and supply ship Var. The Indian side will be led by Rear Admiral Ajay Kochhar,
Nau Sena Medal, Flag Officer Commanding Western Fleet, while the French side will be led by
Rear Admiral Marc Aussedat, Commander Task Force 473.

https://www.indiannavy.nic.in/content/exercise-varuna-2021

6. Consider the following statements regarding Python-5


1. Python is a family of air-to-air missiles (AAMs) built by the Israel.
2. It can carry a 100kg high explosive fragmentation warhead.
3. Python 5 can engage enemy aircraft from very short ranges and near beyond visual range.

Which of the statements given above are correct?


A. 1 and 2 only
B. 2 and 3 only
C. 1 and 3 only
D. 1, 2 and 3

Correct Answer : C

Answer Justification :

Python-5:

www.insightsactivelearn.com 6
Total Marks : 200
Test-30(Subject)
( INSTA Prelims Test Series 2021 )

DRDO conducts maiden trial of Python-5 Air to Air Missile.

Python is a family of air-to-air missiles (AAMs) built by the Israeliweapons


manufacturer Rafael Advanced Defense Systems.

Python 5 can engage enemy aircraft from very short ranges and near beyond
visual range.

Features of the Israeli air-to-air missile

“Python-5 is the most accurate and reliable AAM of the Israeli Air Force and one of the most
sophisticated guided missiles.”

Python-5 incorporates the aerodynamic airframe of the Python-4 missile. It also retains the
inertial navigation system (INS), rocket motor, warhead and proximity fuse of its predecessor.
The advanced technologies combined with operationally proven components deliver superior
performance.

Python-5 is a dual use missile suitable for air-to-air and surface-to-air missions. It
integrates a fifth-generation imaging seeker, modern software, advanced infrared counter-
countermeasure (IRCCM) and flight control systems.

Python-5 has a length of 3.1m, wing span of 64cm and diameter of 16cm. The weight of the
missile is 105kg. It can carry an 11kg high explosive fragmentation warhead equipped
with an active laser proximity fuse. Hence, statement 2 is incorrect.

https://economictimes.indiatimes.com/news/defence/drdo-conducts-maiden-trial-of-python-5-air
-to-air-missile-from-lca-tejas/articleshow/82289789.cms?from=mdr

7. Consider the following statements regarding water use in India


1. 43% of the villages in which farmers plant a winter crop are located in critically water-
depleted regions.
2. The cropping intensity or the amount of land planted in the winter season may decrease by up
to 20% by 2025.

Which of the statements given above is/are correct?


A. 1 only
B. 2 only
C. Both 1 and 2
D. Neither 1 nor 2

www.insightsactivelearn.com 7
Total Marks : 200
Test-30(Subject)
( INSTA Prelims Test Series 2021 )

Correct Answer : B

Answer Justification :

India’s Irrigation system:

1. India is the second-largest producer of wheat in the world, with over 30 million
hectares in the country dedicated to producing this crop.

2. But with severe groundwater depletion, the cropping intensity or the amount of
land planted in the winter season may decrease by up to 20% by 2025, notes a
new paper.

26
3. Some of the important winter crops are wheat, barley, mustard and peas.

m
4. The international team studied India’s three main irrigation types on winter
o
l.c
cropped areas: dug wells, tube wells, canals, and also analysed the groundwater
data from the Central Ground Water Board.

ti 5
5. They found that 13% of the villages in which farmers plant a winter crop are located in
a r
critically water-depleted regions.Hence, statement 1 is incorrect.
h
am
6. The team writes that these villages may lose 68% of their cropped area in future if
i -
access to all groundwater irrigation is lost.
t
n
7. The results suggest that these losses will largely occur in northwest and central India.

www.insightsactivelearn.com 8
Total Marks : 200
Test-30(Subject)
( INSTA Prelims Test Series 2021 )

8. Consider the following statements regarding pandemic


1. The Spanish flu pandemic occurred in 1988-89.
2. The Spanish flu was highly contagious and it was unusually lethal for young, “prime-age”
adults, especially men.

Which of the statements given above is/are correct?


A. 1 only
B. 2 only
C. Both 1 and 2
D. Neither 1 nor 2

www.insightsactivelearn.com 9
Total Marks : 200
Test-30(Subject)
( INSTA Prelims Test Series 2021 )

Correct Answer : B

Answer Justification :

Given the uncertainty and potency of the COVID-19 virus, it was prudent to learn from any
earlier experience. The Spanish flu pandemic of 1918-19, was one of the deadliest in world
history with peak of worldwide mortality in modern times, as it infected around 500 million
persons, or about one-third of the world's population, and killed anywhere from 50 to 100
million people (Barro et al, 2020). Hence, statement 1 is incorrect.

Like COVID-19, the Spanish flu was highly contagious; it was also unusually lethal for
young, “prime-age” adults, especially men. It came in three waves beginning in the spring
of 1918. The second wave, in the fall of 1918, was the largest by far in terms of total infections
and deaths. A third wave occurred in the spring of 1919. The pandemic began during World
War I, and the virus is thought to have been introduced and spread throughout the United
States by soldiers returning from Europe. Lockdowns implemented in 1918 resemble many of
the policies used to reduce the spread of COVID-19, including school, theater, and church
closures, public gathering and funeral bans, quarantine of suspected cases, and restricted
business hours. Other public health interventions used were emphasis on hand-washing,
sanitization practices and social/ physical distancing.

9. Consider the following statements regarding germs


1. The germ theory was proposed by Ignaz Semmelweis in 1885.
2. Louis Pasteur is considered the father of hand hygiene and infection control in hospitals.

Which of the statements given above is/are correct?


A. 1 only
B. 2 only
C. Both 1 and 2
D. Neither 1 nor 2

Correct Answer : D

Answer Justification :

Due to COVID-19, handwashing received attention once more after nearly 170 years. It may be
unbelievable today, but nearly 200 years ago, doctors did not wear gloves for surgeries and
the concept of germs was not known. The germ theory was proposed by Louis Pasteur in
1885.Hence, statement 1 is incorrect.

It all started when a young Hungarian physician Ignaz Semmelweis in the obstetrics
department of Vienna Hospital is 1846 found, to his surprise, that the mortality rate of his
division was sevenfold higher than that of another obstetrics division staffed exclusively by
midwives. Upon further investigation, he found that the physicians would start their day by
conducting autopsies and then proceeding to labour rooms for conducting deliveries, without

www.insightsactivelearn.com 10
Total Marks : 200
Test-30(Subject)
( INSTA Prelims Test Series 2021 )

cleaning their hands. The nurses and midwives, on the other hand, started their days with
deliveries. He then introduced a handwashing policy for all physicians and medical students
before they entered the labour room, and within a year, the mortality was brought down to
one-sixth of the former number. This was the first scientific proof that handwashing helped in
preventing infection, though this did not immediately become popular among doctors. Today,
Ignaz Semmelweis is considered the father of hand hygiene and infection control in
hospitals. Hence, statement 2 is incorrect.

10. Consider the following statements regarding deficits


1. Primary Deficit is the difference between the Revenue Receipts plus Non-Debt Capital
Receipts (NDCR) and the total expenditure.
2. Effective Revenue Deficit is the difference between Revenue Deficit and Grants for Creation of
Capital Assets.

Which of the statements given above is/are correct?


A. 1 only
B. 2 only
C. Both 1 and 2
D. Neither 1 nor 2

Correct Answer : B

Answer Justification :

Fiscal Deficit is the difference between the Revenue Receipts plus Non-Debt Capital
Receipts (NDCR) and the total expenditure.Hence, statement 1 is incorrect.

FD is reflective of the total borrowing requirement of Government. Revenue Deficit refers to


the excess of revenue expenditure over revenue receipts. Effective Revenue Deficit is the
difference between Revenue Deficit and Grants for Creation of Capital Assets. Primary
Deficit is measured as Fiscal Deficit less interest payments.

11. Which of the following taxes form the largest part in receipts under Budget 2021-22?

A. Corporation Tax
B. Income Tax
C. GST
D. Union excise duties

Correct Answer : C

Answer Justification :

www.insightsactivelearn.com 11
Total Marks : 200
Test-30(Subject)
( INSTA Prelims Test Series 2021 )

Rupee Comes From

(Budget 2021-22)

Hence, option (c) is correct.

12. Which of the following forms the largest part in expenditure under Budget 2021-22?

A. Centrally Sponsored schemes


B. Centrally Sponsored schemes
C. Pensions
D. Interest Payments

Correct Answer : D

Answer Justification :

Rupee Goes To

(Budget 2021-22)

www.insightsactivelearn.com 12
Total Marks : 200
Test-30(Subject)
( INSTA Prelims Test Series 2021 )

Hence, option (d) is correct.

13. Arrange the following deficits (In terms of % of GDP) of India from highest to lowest?
1. Fiscal Deficit
2. Primary Deficit
3. Revenue Deficit
4. Effective Revenue Deficit

Select the correct answer using the code given below:


A. 1 3 4 2
B. 2 1 4 3
C. 2 4 1 3
D. 1 2 3 4

Correct Answer : A

Answer Justification :

www.insightsactivelearn.com 13
Total Marks : 200
Test-30(Subject)
( INSTA Prelims Test Series 2021 )

Hence, option (a) is correct.

14. Which of the following forms the largest source of financing fiscal deficit?

A. Market Borrowings
B. Securities against small savings
C. State provident funds
D. External Debts

Correct Answer : A

Answer Justification :

www.insightsactivelearn.com 14
Total Marks : 200
Test-30(Subject)
( INSTA Prelims Test Series 2021 )

Hence, option (a) is correct.

15. Which of the following are part of revenue receipts?


1. Taxes
2. Interest receipts
3. External Grants
4. Recoveries of loans

Select the correct answer using the code given below:


A. 1, 2 and 3 only
B. 2, 3 and 4 only
C. 1, 2 and 4 only
D. 1, 2, 3 and 4

www.insightsactivelearn.com 15
Total Marks : 200
Test-30(Subject)
( INSTA Prelims Test Series 2021 )

Correct Answer : A

Answer Justification :

Hence, statement 4 is incorrect.

16. Consider the following statements regarding trend in tax receipts


1. Tax to GDP ratio for the past decade has been around 15% of GDP.
2. For the last 10 years, direct tax receipt has been higher than indirect tax receipts.

Which of the statements given above is/are correct?


A. 1 only
B. 2 only
C. Both 1 and 2
D. Neither 1 nor 2

www.insightsactivelearn.com 16
Total Marks : 200
Test-30(Subject)
( INSTA Prelims Test Series 2021 )

Correct Answer : D

Answer Justification :

Hence, both statement 1 and 2 are incorrect.

17. Arrange the following subsidies from highest to lowest based on budgetary allocation
1. Fertilizer
2. Food
3. Petroleum

Select the correct answer using the code given below:


A. 1 2 3
B. 1 3 2
C. 2 1 3
D. 2 3 1

Correct Answer : C

Answer Justification :

www.insightsactivelearn.com 17
Total Marks : 200
Test-30(Subject)
( INSTA Prelims Test Series 2021 )

Hence, option (c) is correct.

18. Consider the following statements regarding trend of expenditure in India


1. Agriculture and allied activities has higher expenditure when compared to Rural development.
2. For the past 5 years, the expenditure on health has been increasing.

Which of the statements given above is/are correct?


A. 1 only
B. 2 only
C. Both 1 and 2
D. Neither 1 nor 2

Correct Answer : B

Answer Justification :

Hence, statement 1 is incorrect.

www.insightsactivelearn.com 18
Total Marks : 200
Test-30(Subject)
( INSTA Prelims Test Series 2021 )

19. Which of the following are central sector schemes?


1. Pradhan Mantri Kisan Samman Nidhi
2. Price Stabilization Fund
3. Regional connectivity scheme
4. North East Road Sector Development Scheme

Select the correct answer using the code given below:


A. 1, 2 and 3 only
B. 2, 3 and 4 only
C. 1, 2 and 4 only
D. 1, 2, 3 and 4

Correct Answer : D

Answer Justification :

All the above statements are correct.

www.insightsactivelearn.com 19
Total Marks : 200
Test-30(Subject)
( INSTA Prelims Test Series 2021 )

20. Which of the following are the interventions under Prime Minister Atmanirbhar Swasth Bharat Yojana’
(PMASBY)?
1. Setting up of Integrated Public Health Labs in all districts.
2. Establishing around 11,000 urban Health and Wellness Centres in all the States.
3. Support for around 17,700 rural Health and Wellness Centres in 10 High Focus States.

Which of the statements given above is/are correct?


A. 1 only
B. 2 and 3 only
C. 3 only

www.insightsactivelearn.com 20
Total Marks : 200
Test-30(Subject)
( INSTA Prelims Test Series 2021 )

D. 1, 2 and 3

Correct Answer : D

Answer Justification :

All the above statements are correct.

In the Budget speech of FY 21-22, ‘Prime Minister AtmanirbharSwasth Bharat Yojana’


(PMASBY) scheme has been announced on 1st February, 2021, with an outlay of about
Rs.64,180 Cr over six years (till FY 25-26).This will be in addition to the National Health
Mission.

The main interventions under the scheme to be achieved by FY 25-26 are:

1. Support for 17,788 rural Health and Wellness Centres in 10 High Focus States

2. Establishing 11,024 urban Health and Wellness Centres in all the States.

3. Setting up of Integrated Public Health Labs in all districts and 3382 Block Public
Health Units in11 High Focus states;

4. Establishing Critical Care Hospital Blocks in 602 districts and 12 Central Institutions;

5. Strengthening of the National Centre for Disease Control (NCDC), its 5 regional
branches and 20 metropolitan health surveillance units;

6. Expansion of the Integrated Health Information Portal to all States/UTs to connect all
public health labs;

21. Which of the following are features of National Commission for Allied and Healthcare Professions Bill?
1. An ‘allied health professional’ includes a scientist, therapist, or any other professional who
studies, advises, researches, supervises, or provides preventive, curative, rehabilitative,
therapeutic, or promotional health services.
2. The duration of the degree /diploma for allied health professional should be at least 5,000
hours (over a period of two to four years).

Which of the statements given above is/are correct?


A. 1 only
B. 2 only
C. Both 1 and 2
www.insightsactivelearn.com 21
Total Marks : 200
Test-30(Subject)
( INSTA Prelims Test Series 2021 )

D. Neither 1 nor 2

Correct Answer : D

Answer Justification :

The National Commission for Allied and Healthcare Professions Bill, 2020was introduced
in Rajya Sabha by the Minister of Health and Family Welfare, Dr. Harsh Vardhan, on
September 15, 2020. The Bill seeks to regulate and standardise the education and
practice of allied and healthcare professionals. Key features of the Bill include:

Allied health professional: The Bill defines ‘allied health professional’ as an


associate, technician, or technologist trained to support the diagnosis and
treatment of any illness, disease, injury, or impairment. Such a professional should
have obtained a diploma or degree under this Bill. The duration of the degree
/diploma should be at least 2,000 hours (over a period of two to four years).

Healthcare professional: A ‘healthcare professional’ includes a scientist,


therapist, or any other professional who studies, advises, researches, supervises,
or provides preventive, curative, rehabilitative, therapeutic, or promotional
health services. Such a professional should have obtained a degree under this Bill.
The duration of the degree should be at least 3,600 hours (over a period of three to six
years).

Allied and healthcare professions: The Bill specifies certain categories of allied and
healthcare professions as recognised categories. These are mentioned in the Schedule
to the Bill and include life science professionals, trauma and burn care professionals,
surgical and anaesthesia related technology professionals, physiotherapists, and
nutrition science professionals. The central government may amend this Schedule after
consultation with the National Commission for Allied and Healthcare Profession.

Hence, both statement 1 and 2 are incorrect.

22. Under Budget 2021-22, MITRA scheme was announced for

A. Textile sector
B. ASHA workers
C. Frontline workers
D. Medical devices manufacturers

www.insightsactivelearn.com 22
Total Marks : 200
Test-30(Subject)
( INSTA Prelims Test Series 2021 )

Correct Answer : A

Answer Justification :

Govt. announces launch of Mega Investment Textiles Parks (MITRA) scheme to make
Indian textile industry globally competitive

The Government has proposed a scheme of Mega Investment Textiles Parks (MITRA)
to enable the textile industry to become globally competitive, attract large
investments, boost employment generation and exports. Presenting the Union Budget
2021-22 in the parliament today, Union Finance and Corporate Affairs Minister Smt. Nirmala
Sitharaman said that this will create world class infrastructure with plug and play facilities to
enable create global champions in exports. MITRA will be launched in addition to the
Production Linked Incentive Scheme (PLI).

Hence, option (a) is correct.

www.insightsactivelearn.com 23
Total Marks : 200
Test-30(Subject)
( INSTA Prelims Test Series 2021 )

23. Which of the following are objectives of draft National Rail Plan (NRP)?
1. To increase the modal share of Railways from 45% currently to 65% in freight by 2030.
2. Reduce transit time of freight substantially by increasing average speed of freight trains from
present 22Kmph to 50Kmph.
3. Reduce overall cost of Rail transportation by nearly 30%.

Which of the statements given above is/are correct?


A. 1 only
B. 2 and 3 only
C. 3 only
D. 1, 2 and 3

Correct Answer : B

Answer Justification :

National Rail Plan (NRP):

Context:

The Government has issued the Draft Final Report of the National Rail Plan.

The Plan aims at providing a long-term perspective planning for augmenting the Railway
Network.

Objectives of the plan:

1. To create capacity ahead of demand by 2030, which in turn would cater to growth in
demand right up to 2050.

2. To increase the modal share of Railways from 27% currently to 45% in freight by
2030 as part of a national commitment to reduce Carbon emission and to continue to

www.insightsactivelearn.com 24
Total Marks : 200
Test-30(Subject)
( INSTA Prelims Test Series 2021 )

sustain it. Hence, statement 1 is incorrect.

3. To assess the actual demand in freight and passenger sectors, a yearlong survey was
conducted over hundred representative locations by survey teams spread all over the
country.

4. Forecast growth of traffic in both freight and passenger year on year up to 2030 and on
a decadal basis up to 2050.

5. Formulate strategies based on both operational capacities and commercial policy


initiatives to increase modal share of the Railways in freight to 45% by 2030.

6. Reduce transit time of freight substantially by increasing average speed of


freight trains from present 22Kmph to 50Kmph.

7. Reduce overall cost of Rail transportation by nearly 30% and pass on the benefits
to the customers.

24. Which of the following indicators are measured under Logistics Performance Index (LPI)?
1. Customs
2. Timeliness
3. Ease of International shipment
4. Infrastructure

Select the correct answer using the code given below:


A. 1, 2 and 3 only
B. 2, 3 and 4 only
C. 1, 2 and 4 only
D. 1, 2, 3 and 4

Correct Answer : D

Answer Justification :

All the above statements are correct.

www.insightsactivelearn.com 25
Total Marks : 200
Test-30(Subject)
( INSTA Prelims Test Series 2021 )

www.insightsactivelearn.com 26
Total Marks : 200
Test-30(Subject)
( INSTA Prelims Test Series 2021 )

The Logistics Performance Index (LPI) is an interactive benchmarking tool created by the
World Bank to help countries identify the challenges and opportunities they face in their
performance on trade logistics and what they can do to improve their performance.

Logistics comprises a network of services that supports the physical movement of goods within
and across borders. It is a $4.3 trillion industry. The 2018 Logistics Performance Index (LPI)
scores countries on how efficiently they move goods across and within borders.

https://www.worldbank.org/en/news/infographic/2018/07/24/logistics-performance-index-2018

25. Consider the following statements regarding National Monetization Pipeline


1. It is applicable to both greenfield and brownfield projects.
2. Railways will monetize Dedicated Freight Corridor assets for operations and maintenance,
after commissioning.

Which of the statements given above is/are correct?


A. 1 only
B. 2 only
C. Both 1 and 2
D. Neither 1 nor 2

Correct Answer : B

Answer Justification :

Asset Monetization

The Union Finance Minister announced launch of a “National Monetization Pipeline”


of potential brownfield infrastructure assets stating that Monetizing operating public
infrastructure assets is a very important financing option for new infrastructure
construction.Hence, statement 1 is incorrect.

The Minister informed that an Asset Monetization dashboard will also be created for
tracking the progress and to provide visibility to investors. Some important measures in
the direction of monetization are as follows:

1. National Highways Authority of India and PGCIL each have sponsored one InvIT that will
attract international and domestic institutional investors. Five operational roads with an
estimated enterprise value of Rs.5,000 crore are being transferred to the NHAI InvIT.
Similarly, transmission assets of a value of Rs. 7,000 crore will be transferred to the
PGCIL InvIT.

www.insightsactivelearn.com 27
Total Marks : 200
Test-30(Subject)
( INSTA Prelims Test Series 2021 )

2. Railways will monetize Dedicated Freight Corridor assets for operations and
maintenance, after commissioning.

3. The next lot of Airports will be monetised for operations and management concession.

4. Other core infrastructure assets that will be rolled out under the Asset Monetization
Programme are: (i) NHAI Operational Toll Roads (ii) Transmission Assets of PGCIL (iii)
Oil and Gas Pipelines of GAIL, IOCL and HPCL (iv) AAI Airports in Tier II and III cities,
(v) Other Railway Infrastructure Assets (vi) Warehousing Assets of CPSEs such as
Central Warehousing Corporation and NAFED among others and (vii) Sports Stadiums.

26. Consider the following statements regarding SVAMITVA Scheme


1. It is a centrally sponsored scheme under Ministry of Panchayat Raj.
2. The scheme aims to provide an integrated property validation solution for rural India.
3. Survey of India shall work as the technology partner for implementation.

Which of the statements given above are correct?


A. 1 and 2 only
B. 2 and 3 only
C. 1 and 3 only
D. 1, 2 and 3

Correct Answer : B

Answer Justification :

SVAMITVA Schemeis a Central Sector scheme launched by Hon’ble Prime Minister of


India on National Panchayat Day i.e 24th April 2020. Hence, statement 1 is incorrect.

The Ministry of Panchayati Raj (MoPR) is the Nodal Ministry for implementation of
the scheme. In the States, the Revenue Department / Land Records Department will
be the Nodal Department and shall carry out the scheme with support of State
Panchayati Raj Department.Survey of India shall work as the technology partner for
implementation.

The scheme aims to provide an integrated property validation solution for rural India.
The demarcation of rural abadi areas would be done using Drone Surveying technology. This
would provide the ‘record of rights’ to village household owners possessing houses in
inhabited rural areas in villages which, in turn, would enable them to use their property as a
financial asset for taking loans and other financial benefits from Bank.

www.insightsactivelearn.com 28
Total Marks : 200
Test-30(Subject)
( INSTA Prelims Test Series 2021 )

The scheme seeks to achieve the following objectives: -

1. To bring financial stability to the citizens in rural India by enabling them to use their
property as a financial asset for taking loans and other financial benefits.

2. Creation of accurate land records for rural planning.

3. Determination of property tax, which would accrue to the GPs directly in States where it
is devolved or else, add to the State exchequer.

4. Creation of survey infrastructure and GIS maps that can be leveraged by any department
for their use.

5. To support in preparation of better-quality Gram Panchayat Development Plan (GPDP) by


making use of GIS maps.

6. To reduce property related disputes and legal cases

27. Consider the following statements regarding Research in India


1. The total investment in research and innovation has increased from 0.69 per cent of GDP in
2008 to 0.84 per cent in 2014.
2. Less than 0.5 per cent of Indian students pursue PhD or equivalent level of education.

Which of the statements given above is/are correct?


A. 1 only
B. 2 only
C. Both 1 and 2
D. Neither 1 nor 2

Correct Answer : B

Answer Justification :

What is National Research Foundation?

The total investment in research and innovation has declined from 0.84 per cent of
GDP in 2008 to 0.69 per cent in 2014. Less than 0.5 per cent of Indian students
pursue PhD or equivalent level of education, yet it is not the only issue the number of

www.insightsactivelearn.com 29
Total Marks : 200
Test-30(Subject)
( INSTA Prelims Test Series 2021 )

students pursuing research is also less in India. Hence, statement 1 is incorrect.

The funds allocated to research have declined from 0.84 per cent of GDP in 2008 to 0.69 per
cent in 2014, as mentioned in the draft NEP. The new national education policy also
acknowledges, “research and innovation investment in India is, at the current time, only 0.69
per cent of GDP as compared to 2.8 per cent in the United States of America, 4.3 per cent in
Israel and 4.2 per cent in South Korea.”

In his address at the 106th Indian Science Congress on 3rd January 2019, Honorable Prime
Minister, Shri Narendra Modi, emphasized the need for establishing an expanded research
ecosystem. He said that our national labs, Central Universities, IITs, IISc, TIFR, and IISERs
form the backbone of our research and development. He emphasized that a strong research
ecosystem must be developed in these State universities and colleges also since over 95% of
our students go there.

Come 2021 and an outlay of 50,000 crores over five years was announced in Union Budget
2021-22 for National Research Foundation (NRF).

The Principal Scientific Adviser to the Government of India, Prof. K. VijayRaghavan applauded
this announcement. He pointed out that the National Research Foundation will take research
across the various disciplines.

On 3rd March 2021, while addressing a webinar on effective implementation of Union Budget
2021 provisions, Prime Minister said, “National Research Foundation is being built for the first
time in the country. 50 thousand crore rupees have been allocated for this. This will
strengthen the governance structure of the research related institutions and will improve
linkages between R&D, academia and industry,” on the significance of the establishment of
NRF.

The NRF was formulated, to make the pursuit of science, humanities, art & culture accessible
and available in the language one is most comfortable with. The draft NRF is available here in
12 major regional languages.

28. Consider the following statements regarding Vehicle Scrapping Policy


1. As per the policy government commercial vehicles more than 10 years old and private vehicles
which are over 15 years old will be scrapped.
2. The state governments may be advised to offer a road-tax rebate of up to 50% for personal
vehicles.
3. Vehicle manufacturers will give a discount of 5% to people who will produce the 'Scrapping
Certificate'.

Which of the statements given above is/are correct?

www.insightsactivelearn.com 30
Total Marks : 200
Test-30(Subject)
( INSTA Prelims Test Series 2021 )

A. 1 only
B. 2 and 3 only
C. 3 only
D. 1, 2 and 3

Correct Answer : C

Answer Justification :

Salient features of the new Vehicle Scrapping Policy

The central government will develop a website for single-window clearance, on which
the applicant willing to set up an RSVF will apply with documents and fee. State
governments are to clear the proposals in 60 days.

Key Points

Aim:

Reducing the population of old and defective vehicles, bringing down vehicular air
pollutants, improving road and vehicular safety.

Provisions:

Fitness Test:

Old vehicles will have to pass a fitness test before re-registration and as
per the policy government commercial vehicles more than 15 years old
and private vehicles which are over 20 years old will be scrapped.
Hence, statement 1 is incorrect.

Old vehicles will be tested at the Automated Fitness Center and the fitness
test of the vehicles will be conducted according to international standards.

Emission test, braking system, safety components will be tested and


the vehicles which fail in the fitness test will be scraped.

The Ministry has also issued rules for registration procedure for
scrapping facilities, their powers, and scrapping procedure to be
followed.

www.insightsactivelearn.com 31
Total Marks : 200
Test-30(Subject)
( INSTA Prelims Test Series 2021 )

Road Tax Rebate:

The state governments may be advised to offer a road-tax rebate of up to


25% for personal vehicles and up to 15% for commercial vehicles to
provide incentive to owners of old vehicles to scrap old and unfit vehicles.
Hence, statement 2 is incorrect.

Vehicle Discount:

Vehicle manufacturers will also give a discount of 5% to people who


will produce the 'Scrapping Certificate'and registration fees will be waived
off on the purchase of a new vehicle.

Disincentive:

As a disincentive, increased re-registration fees would be applicable for


vehicles 15 years or older from the initial date registration.

29. Consider the following statements regarding Hydrogen


1. At standard temperature and pressure, hydrogen is a nontoxic, nonmetallic, odorless,
tasteless, colorless, and highly combustible diatomic gas.
2. Since the weight of hydrogen is less than air, it rises in the atmosphere and is therefore
generally found in its pure form, H2.

Which of the statements given above is/are correct?


A. 1 only
B. 2 only
C. Both 1 and 2
D. Neither 1 nor 2

Correct Answer : A

Answer Justification :

National Hydrogen Energy Mission:

Focus on generation of hydrogen from green power resources.

To link India’s growing renewable capacity with the hydrogen economy.

www.insightsactivelearn.com 32
Total Marks : 200
Test-30(Subject)
( INSTA Prelims Test Series 2021 )

India's ambitious goal of 175 GW by 2022 got an impetus in the 2021-22 budget
which allocated Rs. 1500 crore for renewable energy development and NHM.

The usage of hydrogen will not only help India in achieving its emission goals
under the Paris Agreement, but will also reduce import dependency on fossil
fuels.

 Hydrogen:

Hydrogen is the lightest and first element on the periodic table.Since the weight
of hydrogen is less than air, it rises in the atmosphere and is therefore rarely
found in its pure form, H2.Hence, statement 2 is incorrect.

At standard temperature and pressure, hydrogen is a nontoxic, nonmetallic,


odorless, tasteless, colorless, and highly combustible diatomic gas.

Hydrogen fuel is a zero-emission fuel burned with oxygen. It can be used in fuel cells
or internal combustion engines. It is also used as a fuel for spacecraft propulsion.

30. Consider the following statements regarding Cess


1. Cess is a kind of special-purpose tax which is levied over and above basic tax rates.
2. Drawing power from Articles 270 and 271 of the Constitution, the Centre collects cess and
deposits it in the Consolidated Fund of India.

Which of the statements given above is/are correct?


A. 1 only
B. 2 only
C. Both 1 and 2
D. Neither 1 nor 2

Correct Answer : C

Answer Justification :

The Budget for 2021-22 may have refrained from tinkering with your taxes, but it has
proposed a new levy — Agriculture Infrastructure Development Cess.

What is it?

Cess is a kind of special-purpose tax which is levied over and above basic tax rates.
The purpose of the new AIDC is to raise funds to finance spending on developing agriculture

www.insightsactivelearn.com 33
Total Marks : 200
Test-30(Subject)
( INSTA Prelims Test Series 2021 )

infrastructure. Considering that not much private investment is forthcoming for agriculture,
the Centre now seeks to raise a dedicated fund to meet these expenses.

In her Budget speech, the Finance Minister proposed AIDC on a small number of items from
February 2. While applying this cess though, she has made sure that there will not be
additional burden on consumers on most items.

The new cess will be levied on 29 products, prominent among which are gold, silver, imported
apple, imported alcohol (excluding beer), imported pulses, imported palm oil, imported urea,
and petrol/diesel including branded ones. While Basic Custom Duty (BCD) has been lowered
on 25 of these products, Basic Excise Duty (BED) and Special Additional Excise Duty (SAED)
have been lowered on unbranded and branded petrol-diesel. The new cess will only offset the
reduction in customs or excise duty and thus will not raise the tax incidence for consumers.

Drawing power from Articles 270 and 271 of the Constitution, the Centre collects cess
and deposits it in the Consolidated Fund of India. However, the money is then supposed
to be transferred to a segregated fund to be used for specific purpose.

All the above statements are correct.

31. Consider the following statements regarding fiscal policy


1. Counter-cyclical fiscal policy stabilizes the business cycle by being contractionary (reduce
spending/increase taxes) in good times.
2. A pro-cyclical fiscal policy is the one wherein fiscal policy reinforces the business cycle by
being expansionary during good times.

Which of the statements given above is/are correct?


A. 1 only
B. 2 only
C. Both 1 and 2
D. Neither 1 nor 2

Correct Answer : C

Answer Justification :

All the above statements are correct.

www.insightsactivelearn.com 34
Total Marks : 200
Test-30(Subject)
( INSTA Prelims Test Series 2021 )

32. Consider the following statements regarding Government’s debt


1. Floating rate debt of Central Government is less than 2.5 per cent of public debt.
2. Of the total public debt, 70 per cent is held by the Centre.

Which of the statements given above is/are correct?


A. 1 only
B. 2 only
C. Both 1 and 2
D. Neither 1 nor 2

Correct Answer : B

Answer Justification :

The Government’s debt portfolio is characterized by very low foreign exchange risk as
the external debt is only 2.7 per cent of GDP (5.9 per cent of total Central
Government liabilities) (Figure 20). Of the total public debt, 70 per cent is held by the
Centre (Figure 19).

As the central government is entrusted with the responsibility of macro-economic


management, this distribution of debt between the centre and states is desirable because of
the incentive compatibility that it generates. The long maturity profile of India’s public debt
(issuance of longer tenure bonds) along with a small share of floating rate debt (floating rate
debt of Central Government is less than 5 per cent of public debt) tends to limit rollover
risks, and insulates the debt portfolio from interest rate volatility (Figure 21 and 22). Hence,
statement 1 is incorrect.

www.insightsactivelearn.com 35
Total Marks : 200
Test-30(Subject)
( INSTA Prelims Test Series 2021 )

33. Consider the following statements regarding credit ratings


1. Currently, India is rated speculative grade by three major CRAs S&P, Moody’s and Fitch.
2. Speculative grade rating indicates a higher likelihood of default on borrowings.

Which of the statements given above is/are correct?


A. 1 only
B. 2 only
C. Both 1 and 2
D. Neither 1 nor 2

Correct Answer : B

Answer Justification :

Currently, India is rated investment grade by three major CRAs – S&P, Moody’s and
Fitch. Hence, statement 1 is incorrect.

India’s sovereign credit ratings during 1998-2020 are presented in Table 1. Rationale given for
the same by these CRAs is depicted in Figure 3. India’s sovereign credit rating downgrades
during 1998-2018 are mainly confined to the 1990s on account of the post-Pokhran sanctions
in 1998. India’s sovereign credit ratings upgrades have mainly been witnessed in the second
half of 2000s, in recognition of higher economic growth prospects and strengthened
fundamentals of the Indian economy. Further, during most of the 1990s and mid-2000s, India’s
sovereign credit rating was speculative grade.

www.insightsactivelearn.com 36
Total Marks : 200
Test-30(Subject)
( INSTA Prelims Test Series 2021 )

Sovereign credit ratings broadly rate countries as either investment grade or


speculative grade, with the latter projected to have a higher likelihood of default on
borrowings. The threshold of Investment grade is considered to be BBB- for S&P and Fitch
and Baa3 for Moody’s.

34. India’s sovereign credit rating upgrade led to?


1. Sensex return on average increased.
2. Exchange rate (INR/USD) on an average appreciated.
3. Sensex return on average decreased.
4. Exchange rate (INR/USD) on an average depreciated.

Select the correct answer using the code given below:


A. 1 and 2 only
B. 2 and 3 only
C. 3 and 4 only
D. 1 and 4 only

Correct Answer : A

Answer Justification :

Effect of India’s Sovereign Credit Rating Upgrades

3.40 Table 3 summarizes the average changes in select indicators during and after India’s
sovereign credit ratings upgrade between 1998-2018. In the short run, during India’s
sovereign credit rating upgrades, Sensex return on average fell by around 0.7 per cent over
the previous day, and grew at 0.2 per cent over the next two weeks. Exchange rate (INR/USD),
on average, appreciated by around 0.05 per cent over the previous day during the rating
upgrade, and appreciated by 0.03 per cent over the next two weeks.

3.41 Over the medium term, during India’s sovereign credit ratings upgrade, Sensex return on
average rose by around two per cent over the previous month and grew at an average rate of
1.8 per cent over the next six months. Exchange rate (INR/USD), on average, appreciated by
around 0.3 per cent over the previous month during the rating upgrade, and appreciated by
0.4 per cent over the next 6 months. During ratings upgrade, yield on G-Sec (5 year), on
average, increased by 0.2 per cent over the previous month, and grew at 0.6 per cent over the
next six months. Yield on G-Sec (10 year), on average, fell by 0.5 per cent over the previous
month, and grew at an average rate of 0.7 per cent over the next six months. Spread (RHS), on
average, declined by five per cent over the previous month, and grew at an average rate of five
per cent over the next six months (Table 3).

www.insightsactivelearn.com 37
Total Marks : 200
Test-30(Subject)
( INSTA Prelims Test Series 2021 )

In the long term, during India’s sovereign credit ratings upgrade, Sensex return on average
rose by around 36 per cent over the previous year and grew at an average rate of 13 per cent
in the next year. Exchange rate (INR/USD), on average, appreciated by around 1.5 per cent
over the previous year during the rating upgrade, and appreciated by two per cent in the next
year. FPI Equity, on average, increased by 264 per cent over the previous year during the
rating upgrade, and grew by 303 per cent the next year. Average FPI Debt too followed a
similar pattern, increasing by 286 per cent, on average, during the rating upgrades, and grew
at an average rate of 578 per cent the next year (Table 3).

Hence, option (a) is correct.

35. Consider the following statements regarding health of a nation


1. Increasing life expectancy from 50 to 70 years could raise the economic growth rate by 1.4
percentage points per year.
2. An increase in public health expenditure from the current levels in India to 3 per cent of GDP
can reduce the Out-of-pocket(oop) expenditure from 60 per cent currently to about 30 per
cent.

Which of the statements given above is/are correct?


A. 1 only
B. 2 only
C. Both 1 and 2
D. Neither 1 nor 2

Correct Answer : C

Answer Justification :

All the above statements are correct.

The health of a nation depends critically on its citizens having access to an equitable,
affordable and accountable healthcare system. Health affects domestic economic growth
directly through labour productivity and the economic burden of illnesses (WHO 2004).

www.insightsactivelearn.com 38
Total Marks : 200
Test-30(Subject)
( INSTA Prelims Test Series 2021 )

Increasing life expectancy from 50 to 70 years (a 40 per cent increase) could raise the
economic growth rate by 1.4 percentage points per year (WHO 2004).

Increased prioritization of healthcare in the central and state budgets is important as it


crucially impacts how much protection citizens get against financial hardships due to out of-
pocket payments made for healthcare (WHO 2010). OOP for health increase the risk of
vulnerable groups slipping into poverty because of catastrophic health expenditures
(O’Donnell et al. 2007; Berki 1986; van Doorslaer et al. 2006). Figure 3 shows that at low
levels of public health expenditure, i.e., were public healthcare expenditure as a per cent of
GDP is less than 3 per cent, OOP expenditure as a share of total health expenditure drops
precipitously when public health expenditure increases. For instance, an increase in public
health expenditure from the current levels in India to 3 per cent of GDP can reduce
the OOP expenditure from 60 per cent currently to about 30 per cent.

36. Consider the following statements regarding World Rule of Law Index
1. The ‘World Rule of Law Index’ published by the Transparency International.
2. It provides cross country comparison on various aspects of regulatory enforcement.

Which of the statements given above is/are correct?


A. 1 only
B. 2 only
C. Both 1 and 2
D. Neither 1 nor 2

Correct Answer : B

Answer Justification :

The ‘World Rule of Law Index’ published by the World Justice Projectprovides cross
country comparison on various aspects of regulatory enforcement.Hence, statement 1
is incorrect.

The index has various sub-categories, which capture compliance to due processes,
effectiveness, timelines, etc. In 2020, India’s rank is 45 out of 128 countries in the category of
‘Due process is respected in administrative proceedings’ (proxy for following due process). In
contrast, in the category ‘Government regulations are effectively enforced’ (proxy for
regulatory quality/effectiveness), the country’s rank is 104 (Table 1). India stands at 89th rank
in ‘Administrative Proceedings are conducted without unreasonable delay’ (proxy for
timeliness) and 107th in ‘Administrative

Proceedings are applied and enforced without improper influence’ (proxy for rent seeking).
This shows that, contrary to the popular belief, India is relatively good at complying with
processes, but lag in regulatory effectiveness.

www.insightsactivelearn.com 39
Total Marks : 200
Test-30(Subject)
( INSTA Prelims Test Series 2021 )

37. Consider the following statements regarding Faceless Assessment Scheme 2020
1. The Faceless Assessment Scheme, 2019 was based on the idea that automated random
allocation of cases across Income Tax teams.
2. Any order passed outside the scheme shall be invalid.
3. The scheme establishes a National Faceless Assessment Centre (NFAC) headed by Finance
Secretary.

Which of the statements given above are correct?


A. 1 and 2 only
B. 2 and 3 only
C. 1 and 3 only
D. 1, 2 and 3

Correct Answer : A

Answer Justification :

Faceless Assessment Scheme 2020

2.19 The Faceless Assessment Scheme, 2019 (earlier called the e assessment Scheme
and renamed in August 2020) was based on the idea that automated random
allocation of cases across Income Tax teams with dynamic jurisdiction and elimination of
face-to-face contact between the income-tax authorities and the taxpayer can lead to an
efficient, non-discretionary, unbiased single window system of assessment. In 2020, the scope
of Faceless Assessment Scheme 2019 was broadened by bringing all the pending
assessment cases across the country within the purview of the Scheme and declaring
that any order passed outside the scheme shall be invalid.

2.20 The scheme establishes a National Faceless Assessment Centre (NFAC) in Delhi,
headed byPrincipal Chief Commissioner of Income Tax, as the sole point of contact
between the Department and the taxpayer. Hence, statement 3 is incorrect.

www.insightsactivelearn.com 40
Total Marks : 200
Test-30(Subject)
( INSTA Prelims Test Series 2021 )

All notices or communications to and from the taxpayer, and internal communications related
to assessment process within the Department are routed through the NFAC. To further
facilitate and streamline the process of assessment there are various Regional Faceless
Assessment Centers which are vested with the power to make assessments. A schematic
diagram of Faceless Assessment procedure may be seen in figure 7.

IMAGE

38. The concept of taxpayers’ Bill of Rights was first introduced in

A. UK
B. USA
C. Russia
D. France

Correct Answer : B

Answer Justification :

Traditionally tax administrations paid limited attention to taxpayer service while performing
the functions of regulator and enforcer of tax laws. However, due to an increased demand for
better services to the tax payers, there has been a worldwide recognition of the rights of the
tax payers, by publishing formal ‘taxpayers’ charters’ or including behaviour expected from
officials in the revenue body’s ‘mission statement’. For instance, the concept of taxpayers’
Bill of Rights was first introduced in the US in 1988 and the Taxpayer Bill of Rights,
which grouped the existing rights in the tax code into ten clearly defined
fundamental rights and applied to all taxpayers, was adopted by the revenue
authorities in 2014. Hence, option (b) is correct.

The introduction of taxpayer charter by Government of India as a part of the ‘Honoring the
Honest’ platform is, thus, an important step in this direction, as it emphasizes the importance
of fair, courteous and reasonable treatment to taxpayer.

In addition to explicitly recognizing the taxpayers’ rights, presence of a dedicated institution


aimed to take up issues from taxpayers’ perspective helps in developing trust of the taxpayers
in the system. This ensures that the taxpayers understand their rights and are treated fairly.
Thus the next step may be to ensure enforcement of rights.

39. At present, which of the following are tax grievance redressal system available to citizens or organization?

www.insightsactivelearn.com 41
Total Marks : 200
Test-30(Subject)
( INSTA Prelims Test Series 2021 )

1. Aaykar Sewa Kendras (ASK),


2. e-nivaran portal
3. CPGRAMS (Central Public Grievance Redress and Monitoring System)
4. Income Tax Ombudsman

Select the correct answer using the code given below:


A. 1, 2 and 3 only
B. 2, 3 and 4 only
C. 1, 2 and 4 only
D. 1, 2, 3 and 4

Correct Answer : A

Answer Justification :

India’s experience with Tax Ombudsman

• In India, the institution of Income Tax Om budsman was created in 2003 and Indirect Tax
Ombudsman came to existence in 2011. The ombudsman was appointed at the regional offices
by the Central Government from amongst the serving officers, to enquire into
grievances/complaints against the functioning of the tax authority.

• Since the functioning of Ombudsman was governed by the guidelines (Income Tax
Ombudsman Guidelines 2010 and Indirect Tax Ombudsman Guidelines 2011), and there was
no act of law empowering it with different functions, the institution of Ombudsman was
ineffective and the decisions only advisory in nature. The Ombudsman could settle complaints
either through agreements between the complainant and the tax department through
conciliation and mediation or by passing an award, with a token compensation for loss suffered
by the complainant not exceeding ` 5,000.

• The institutions of Ombudsman for direct and indirect taxes were, therefore,
abolished in February 2019. The present tax grievance redressal system consists of
grievance cells headed by department officials/ Aaykar Sewa Kendras (ASK), e-
nivaran portal which is a separate and dedicated window for grievance redressal in
the Income Tax Business Application, and CPGRAMS (Central Public Grievance
Redress and Monitoring System). Hence, statement 4 is incorrect.

40. Consider the following statements regarding Remission of Duties and Taxes on Exported Products (RoDTEP)
1. The RoDTEP rates would be notified by the Department of Revenue.
2. Under this Scheme, duties and taxes levied at the Central, State and local levels will be
refunded to exporters in their ledger account with Customs.

www.insightsactivelearn.com 42
Total Marks : 200
Test-30(Subject)
( INSTA Prelims Test Series 2021 )

Which of the statements given above is/are correct?


A. 1 only
B. 2 only
C. Both 1 and 2
D. Neither 1 nor 2

Correct Answer : B

Answer Justification :

Remission of Duties and Taxes on Exported Products (RoDTEP)

3.37 India's various export promotion schemes including Merchandise Exports from India
Scheme (MEIS), were challenged by the United States in WTO in early 2018. The final report
of the WTO panel observed that MEIS is a "prohibited subsidy" and needs to be withdrawn,
against which an appeal has been filed by India. In order to continue supporting the industry
and to eliminate any uncertainty amongst the exporting community, Government has rolled out
a new WTO compliant scheme, namely Remission of Duties and Taxes on Exported Products
(RoDTEP), for all export goods with effect from 1st January, 2021.

3.38 Under this Scheme, duties and taxes levied at the Central, State and local levels,
such as electricity duties and VAT on fuel used for transportation, which are not
getting exempted or refunded under any other existing mechanism will be refunded
to exporters in their ledger account with Customs.The credits can be used to pay
basic customs duty on imported goods or transferred to other importers – facilitating ease
of transactions for exports. The RoDTEP rates would be notified by the Department of
Commerce. Hence, statement 1 is incorrect.

41. Consider the following statements regarding borrowing limits of states under Atma Nirbhar Bharat package
1. Under the Atma Nirbhar Bharat package, additional borrowing limit of up to 5 percent of
Gross State Domestic Product (GSDP) was allowed to the States.
2. Of the additional 2 per cent borrowing allowed to the States, the first instalment of 0.5 per
cent borrowing was untied for all the states.

Which of the statements given above is/are correct?


A. 1 only
B. 2 only
C. Both 1 and 2
D. Neither 1 nor 2

Correct Answer : B

www.insightsactivelearn.com 43
Total Marks : 200
Test-30(Subject)
( INSTA Prelims Test Series 2021 )

Answer Justification :

Enhanced limit of borrowing for FY2020-21 under Atma Nirbhar Bharat package

• Under the Atma Nirbhar Bharat package, additional borrowing limit of up to 2


percent of Gross State Domestic Product (GSDP) was allowed to the States, which was
equivalent to ` 4.27 lakh crore. Hence, statement 1 is incorrect.

Of the additional 2 per cent borrowing allowed to the States, the first instalment of
0.5 per cent borrowing was untied for all the states. The second part amounting to 1 per
cent of GSDP was subject to implementation of following four specific State level reforms,
where weightage of each reform is 0.25 per cent of GSDP:-

a) Implementation of One Nation One Ration Card System;

b) Ease of doing business reform;

c) Urban Local body/ utility reforms; and

d) Power Sector reforms

The final 0.5 per cent borrowing was conditional on undertaking at least 3 out of the above-
mentioned reforms.

42. Consider the following statements regarding State Disaster Relief Fund
1. The Central Government had decided to treat COVID-19 as a notified disaster for the purpose
of providing assistance under SDRF.
2. It is created under Disaster management act 2005.

Which of the statements given above is/are correct?


A. 1 only
B. 2 only
C. Both 1 and 2
D. Neither 1 nor 2

Correct Answer : C

Answer Justification :

All the above statements are correct.

SDRF

• The Central Government by way of a special one-time dispensation had decided to


treat COVID-19 as a notified disaster for the purpose of providing assistance under
SDRF. To strengthen the States to deal with the pandemic, the Centre had released the 1st

www.insightsactivelearn.com 44
Total Marks : 200
Test-30(Subject)
( INSTA Prelims Test Series 2021 )

instalment of SDRF amounting to ` 11,092 crore to State Governments in April 2020. In


September 2020, the states’ limit for spending the SDRF during FY 2020-21 was raised to
50%, in order to support them in containment measures of COVID-19 including measures for
quarantine, sample collection and screening; and procurement of essential equipment/ labs for
response to COVID-19.

43. Global Financial Stability Report 2020 is published by

A. IMF
B. World Bank
C. World Economic Forum
D. Bank for International Settlement

Correct Answer : A

Answer Justification :

Global Financial Stability Report 2020

As per IMF’s October Global Financial Stability Report 2020, near-term global financial
stability risks have been contained for now due to the unprecedented and timely policy
responses to maintain the flow of credit to the economy and avoided adverse macro-financial
feedback loops, thereby creating a bridge to recovery. However, vulnerabilities have increased
in the nonfinancial corporate sector, as firms have taken on more debt to cope with cash
shortages and in the sovereign sector, as fiscal deficits have widened to support the economy.

Hence, option (a) is correct.

44. Which of the following had the largest share in Export commodities in 2020?

A. Drug formulations and biologicals


B. Petroleum products
C. Iron and Steel
D. Organic chemicals

Correct Answer : B

Answer Justification :

www.insightsactivelearn.com 45
Total Marks : 200
Test-30(Subject)
( INSTA Prelims Test Series 2021 )

Hence, option (b) is correct.

45. Consider the following statements regarding Indian pharmaceutical industry


1. India was at 11th position in terms of share in world pharma exports in 2019.
2. Indian pharmaceutical industry is largest in the world (in terms of volume).

Which of the statements given above is/are correct?


A. 1 only
B. 2 only
C. Both 1 and 2
D. Neither 1 nor 2

Correct Answer : A

Answer Justification :

Indian pharmaceutical industry is third largest in the world, in terms of volume,


behind China and Italy and 14th largest in terms of value. Hence, statement 2 is incorrect.

India almost doubled its share in world pharma exports in a span of ten years from 1.4 per
cent in 2010 to 2.6 per cent in 2019. India was at 11th position in terms of share in world
pharma exports in 2019 with Germany, Switzerland and USA occupying the top three
positions.

The global pharmaceutical market is set to exceed US$ 1.5 trillion by 2023. Against this
backdrop, the Indian pharmaceutical industry is currently valued at US$ 41 billion and is
expected to grow to US$ 65 billion by 2024 and about US$ 120-130 billion by 2030. A
significant raw material base and availability of a skilled workforce have enabled India to
emerge as an international manufacturing hub for generic medicines. Further, India is the only
country with largest number of US-FDA compliant pharma plants (more than 262 including
APIs) outside of USA.

www.insightsactivelearn.com 46
Total Marks : 200
Test-30(Subject)
( INSTA Prelims Test Series 2021 )

46. Which of the following country was the largest export market for India in 2020?

A. UAE
B. China
C. USA
D. Bangladesh

Correct Answer : C

Answer Justification :

In so far as the top export destinations are concerned, USA continues to be the largest
export market for India in April-November, 2020, while China has occupied the 2nd
position, moving up from 3rd spot in April-November, 2019. Exports to China in April-
November, 2020 constitute around 7.8 per cent vis-à-vis 5.4 per cent in April-November, 2019
(Figure 7). Malaysia is a new entrant among the top 10 export destinations, as compared to
last year, while Nepal no longer occupies position among the top 10 destinations.

Hence, option (c) is correct.

47. Which of the following sectors are covered under Production-Linked Incentive (PLI) Scheme?
1. Advanced chemistry cell battery
2. Electronic/technology products
3. Automobile and auto component
4. Pharmaceuticals drugs

www.insightsactivelearn.com 47
Total Marks : 200
Test-30(Subject)
( INSTA Prelims Test Series 2021 )

Select the correct answer using the code given below:


A. 1, 2 and 3 only
B. 2, 3 and 4 only
C. 1, 2 and 4 only
D. 1, 2, 3 and 4

Correct Answer : D

Answer Justification :

All the above statements are correct.

Production-Linked Incentive (PLI) Scheme

3.39 In order to boost domestic manufacturing and exports, the Production Linked Incentive
(PLI) scheme with an outlay of `1.46 lakh crore has been introduced. This Scheme aims to give
incentive to companies on incremental sales from products manufactured in domestic units.
The ten-identified champion sectors under PLI scheme are advanced chemistry cell (ACC)
battery (approved financial outlay over a five year period of `18,100 crore),
electronic/technology products (`5,000 crore), automobile and auto component (`57,042
crore), pharmaceuticals drugs (`15,000 crore), telecom and networking products (`12,195
crore), textile products (`10,683 crore), food products (`10,900 crore), high efficiency solar
photovoltaic modules (`4,500 crore), white goods (ACs and LEDs) (`6,238 crore) and specialty
steel (`6,322 crore). These are in addition to the already notified PLI schemes for mobile
manufacturing and specified electronic components (`40,951 crore), critical Key Starting
materials/ Drug Intermediaries and Active Pharmaceutical Ingredients (`6,940 crore) and
manufacturing of medical devices (`7420 crore).

48. Consider the following statements regarding current account deficit


1. India’s current account deficit averaged 2.2 per cent of GDP in the last 10 years.
2. Since 1991, India never had current account surplus.

Which of the statements given above is/are correct?


A. 1 only
B. 2 only
C. Both 1 and 2
D. Neither 1 nor 2

Correct Answer : A

Answer Justification :

India’s current account deficit averaged 2.2 per cent of GDP in the last 10 years.
Reversing this trend, current account balance turned into surplus (0.1 per cent of GDP) in Q4:

www.insightsactivelearn.com 48
Total Marks : 200
Test-30(Subject)
( INSTA Prelims Test Series 2021 )

FY 2019-20 on the back of, among others, a lower trade deficit and a sharp rise in net invisible
receipts. This quarterly surplus was registered after a gap of 13 years after Q4: FY
2006-07. This has been followed by successive current account surpluses in Q1 and
Q2 of FY 2020-21. In H1: FY 2020-21, steep contraction in merchandise imports and lower
outgo for travel services led to a sharper fall in current payments (by 30.8 per cent) than
current receipts (15.1 per cent) – leading to a current account surplus of US$ 34.7 billion (3.1
per cent of GDP) (Figure 13). Given the trend in imports of both goods and services, it is
expected that India will end with an annual current account surplus of at least 2 per cent of
GDP – after a period of 17 years.

Hence, statement 2 is incorrect.

49. Consider the following statements regarding external debt


1. India’s external debt to exports ratio dropped secularly downwards since the crisis year 1992.
2. The China is the most heavily indebted country in the world.

Which of the statements given above is/are correct?


A. 1 only
B. 2 only
C. Both 1 and 2
D. Neither 1 nor 2

Correct Answer : A

Answer Justification :

The total external debt of the world, at US$ 89 trillion as at end-June 2020, grew at lower rate
of 1.0 per cent over the level as at-end March 2020 than that (2.2 per cent) registered a
quarter ago.3 The US is the most heavily indebted country in the world with 23.9 per
cent of the total external debt stock. India is placed at 23rd position globally with an estimated
stock at US$ 554.4 billion as at end-June 2020. Hence, statement 2 is incorrect.

Analysis of maturity profile of India’s external debt as at end-June 2020 among the Special
Data Dissemination Standards (SDDS) reporting countries underscores that India’s share of
short-term debt, at 18.9 per cent, is not only lower than the median share of 24.2 per cent, but
also smaller than that of any top 20 debtor countries. Further, among the SDDS and General
Data Dissemination Standards (GDDS) countries, India’s share of government sector in gross
external debt as at end-June 2020 at 18 per cent is modest and lower than the median share of
at 29.7 per cent.

India’s external debt to exports ratio dropped secularly downwards since the crisis
year 1992, though it has climbed up in the recent years and is now hovering in the close
vicinity of the optimal zone.

www.insightsactivelearn.com 49
Total Marks : 200
Test-30(Subject)
( INSTA Prelims Test Series 2021 )

50. Which of the following country has the largest foreign exchange reserves holder?

A. USA
B. China
C. Japan
D. Switzerland

Correct Answer : B

Answer Justification :

While improved current account balance has been a key factor for reserve accretion in H1 of
2020-21, robust capital flows, particularly FDI and FPI, in subsequent months largely drove
foreign exchange reserves to an all-time high of US$ 586.1 billion as on January 8, 2021,
covering about 18 months of imports (Figure 19). As at end-September 2020, India is the
fifth largest foreign exchange reserves holder among all countries of the world after
China, Japan, Switzerland and Russia. India’s international financial liabilities are 210.7
per cent of foreign exchange reserves as at end September 2020 as compared with 229.7 per
cent as at end-March 2020.

Hence, option (b) is correct.

51. Consider the following statements regarding Sea Buckthorn


1. It is used to produce bio-diesel
2. It is found above the tree line in the Himalayan region.
3. It prevents soil erosion in the region which it grows.

Which of the statements given above is/are correct?


A. 1 and 3 only
B. 2 only
C. 2 and 3 only
D. 1, 2 and 3

www.insightsactivelearn.com 50
Total Marks : 200
Test-30(Subject)
( INSTA Prelims Test Series 2021 )

Correct Answer : D

Answer Justification :

Sea Buckthorn is a shrub which produces an orange-yellow coloured edible berry. It is found
above the tree line in the Himalayan region, generally in dry areas such as the cold deserts of
Ladakh and Spiti. Hence Statement 2 is correct

In Himachal Pradesh, it is locally called chharma and grows in the wild in Lahaul and Spiti and
parts of Kinnaur. A major part is covered by this plant in Himachal Pradesh, Ladakh,
Uttarakhand, Sikkim and Arunachal Pradesh.

Seabuckthorn is a soil-binding plant which prevents soil-erosion, checks siltation in rivers and
helps preserve floral biodiversity. Its fruit and leaves are rich in vitamins, carotenoids and
omega fatty acids and it can help troops in acclimatising to high-altitude. Hence Statement 3
is correct.

Seabuckthorn is used to produce bio-diesel ..http://www.ijetch.org/papers/119--T455.pdf.


Hence Statement 1 is correct

https://indianexpress.com/article/explained/himachal-pradesh-seabuckthorn-project-explained-
7226260/

52. Which of the following is/are the possible impacts of Bamboo flowering in India?
1. It may adversely affect migration, especially by elephants.
2. It may worsen the increasing man-animal conflict.
3. Increasing incidence of Forest Fires

Which of the statements given above is/are correct?


A. 2 only
B. 1 and 3 only
C. 2 and 3 only
D. 1, 2 and 3

Correct Answer : D

Answer Justification :

All the statements given above are correct.

The ‘gregarious flowering of bamboo’ inside the Wayanad Wildlife Sanctuary (WWS) and the
nearby Mudumalai Tiger Reserve and Gudalur forest division in Tamil Nadu may pose a threat
to wildlife in the Nilgiri biosphere, a major tiger and elephant habitat.

www.insightsactivelearn.com 51
Total Marks : 200
Test-30(Subject)
( INSTA Prelims Test Series 2021 )

The bamboo groves in the Wayanad forest are the mainstay of herbivores in the Nilgiri
biosphere during summer. With the advent of the season, migration of wild animals starts from
the adjacent sanctuaries in Karnataka and Tamil Nadu to Wayanad thanks to shortage of
fodder and water.

“The gregarious flowering may adversely affect migration, especially by elephants, wild
gaur, and other lower herbivores owing to the mass destruction of bamboo groves
after the flowering,” S. Narendra Babu, wildlife warden, WWS, said.

“Bamboo groves, which grow in more than 500 hectares of the 344.44 sq.km. of the sanctuary,
have fully bloomed, a phenomenon said to occur once in the life cycle of bamboo plants,” he
added.

Moreover, pointing to a threat to wildlife as well as the ecology of the Nilgiri biosphere, it is
reported that over 25% of bamboo groves in WWS and nearby sanctuaries have bloomed since
2010, and the phenomenon is continuing.

Thorny bamboo (Bamboosa Bambos) is a monocarpic (flowering only once) plant belonging to
the Poaceae family (grass family), and its flowering cycle varies from 40 to 60 years.

Profuse natural regeneration occurs from seeds after gregarious flowering. Seeds have no
dormancy, and it helps germination under favourable conditions soon after seed fall, Mr. Babu
said.

But protection from fire and grazing is essential for proper establishment of seedlings, he
observed.

Fire incidents have been comparatively low in the sanctuary for the past five years
owing to summer rain and conservation measures adopted by the Forest Department.
However, combustible materials have accumulated in the sanctuary this year, and a mere
spark may cause an uncontrollable disaster, he said.

Farmers living near the sanctuary fear that the destruction of bamboo groves may
worsen the increasing man-animal conflict.

https://www.thehindu.com/news/national/kerala/gregarious-bamboo-flowering-in-wws-a-threat-
to-wildlife/article34063077.ece

53. Asola Bhatti Wildlife Sanctuary, sometime seen in the news, is located in

A. Rajasthan
B. Delhi
C. Punjab
D. None of the above

Correct Answer : B

www.insightsactivelearn.com 52
Total Marks : 200
Test-30(Subject)
( INSTA Prelims Test Series 2021 )

Answer Justification :

Asola-Bhati Wildlife Sanctuary covering 32.71 km2 area on the Southern Delhi Ridge
of Aravalli hill range on Delhi-Haryana border lies in Southern Delhi as well as
northern parts of Faridabad and Gurugram districts of Haryana state.

Biodiversity significance of Ridge lies in its merger with Indo-Gangetic plains, as it is the part
of the Northern Aravalli leopard wildlife corridor, an important wildlife corridor which starts
from the Sariska National Park in Rajasthan, passes through Nuh, Faridabad and Gurugram
districts of Haryana and ends at Delhi Ridge

https://indianexpress.com/article/cities/delhi/panel-to-look-into-proposal-on-dumping-inert-wast
e-in-bhatti-mines-7216319/

54. Consider the following statements regarding Matua Community


1. The Matua Mahasangha, a religious reform movement and a sect, was formed by Harichand
Thakur in East Bengal in the mid-1800s.
2. Matua-mahasangha believe in Swayam-Dikshiti through the chanting of Harinaam.

Which of the statements given above is/are correct?


A. 1 only
B. 2 only
C. Both 1 and 2
D. Neither 1 nor 2

Correct Answer : C

Answer Justification :

Both the statements are correct.

Matua Mahasangha or MMS is a religious reformation movement that originated,


around 1860 AD, in what is today Bangladesh, with a considerable number of
adherents both in Bangladesh and in West Bengal in India. Matua is a sect of Vaisnavite
Hinduism who are Namasudras, a Scheduled Caste group.

The movement was launched as a reformation by the followers of Harichand Thakur. Thakur
attained atmadarshan at an early age and would subsequently preach his Darshan in Twelve
Directives. The teachings of Thakur establish education as preeminently important for the
adherent and the upliftment of the population the adherent's duty, while also providing a
formula for ending social conflict.

Matua-mahasangha believe in Swayam-Dikshiti ("Self-Realisation") through the


chanting of Harinaam, i.e., chanting the Holy name of God Hari. Harichand stressed the
congressional chanting of Lord's name kirtan as the sole mean to Mukti. His followers in 19th

www.insightsactivelearn.com 53
Total Marks : 200
Test-30(Subject)
( INSTA Prelims Test Series 2021 )

century were enchanted by congregational chanting of Lord Hari, which in Bengali called
'hariname matuara' giving the sect’s current name 'Matua'. So, anyone who has faith in the
Darshan or Philosophy of God Harichand belongs to the Matua-mahasanhga.

https://indianexpress.com/article/explained/this-word-means-matua-community-5567348/

55. Consider the following statements regarding Millets


1. Tamil Nadu and Rajasthan are the leading states in the production of millets.
2. Millets helps in carbon sequestration.
3. Government provides Minimum Support price for selected millets

Which of the statements given above is/are correct?


A. 1 and 3 only
B. 2 and 3 only
C. 1 and 2 only
D. 1, 2 and 3

Correct Answer : B

Answer Justification :

The three major millet crops currently growing in India are jowar (sorghum), bajra (pearl
millet) and ragi (finger millet).

India currently has around 14 million hectares of land under millets cultivation and produces
around 14 million tonnes a year with Rajasthan, Maharashtra, and Karnataka leading in
millet farming. Hence Statement 1 is incorrect.

The country grows close to a dozen varieties of millets and widely consumed among them
include pearl millet, foxtail millet, sorghum, finger millet, and kodo millet.

Millet crops sequester carbon from the atmosphere. It provides nutritional security and
economic Security.

https://indianexpress.com/article/explained/this-word-means-matua-community-5567348/

56. Nine-Dash Line Dispute, sometime seen in the news, is related to

A. Iran and Israel Conflict


B. North and South Korea Conflict
C. South China Sea Dispute
D. None of the above

www.insightsactivelearn.com 54
Total Marks : 200
Test-30(Subject)
( INSTA Prelims Test Series 2021 )

Correct Answer : C

Answer Justification :

China currently claims ‘indisputable sovereignty’ over the South China Sea, and some
Chinese officials refer to it as the country’s ‘blue national soil’ — a term used to refer
to the country’s offshore waters.

The origins of the nine-dash line can be traced back to the official maps made by the
Nationalist Kuomintang (also known as the Chinese Nationalist Party) government before and
after World War II, according to the book, Asia’s Cauldron, by US scholar and strategist
Robert D. Kaplan.

Eventually, Mao Zedong’s Communist forces defeated the Kuomintang government, who then
fled to Taiwan, establishing their rule there. As a result, both the People’s Republic of China
and Taiwan officially claim ownership over the South China Sea.

The nine-dash line was originally an 11-dash line, and Chinese geographer Yang Huairen
helped etch it. Yang was born in 1917 and pursued his education in the UK before being
employed by China’s Nationalist government.

“In 1947, he worked on the map introducing the 11-dash line and 286 bits of rock and
turf in the South China Sea. Yang helped to officially name each chunk of rock and
reef, referring to the territory collectively as the ‘South China Sea Islands’,” noted a
feature in Time magazine.

In 1949, when the nationalist government lost to the Communists, and relocated to Taiwan,
Yang stayed back. However, he was eventually persecuted as an “anti-revolutionary academic
authority” during the Cultural Revolution — a sociopolitical movement in China from 1966 to
1976 — for his role in the Kuomintang government.

https://theprint.in/theprint-essential/what-is-nine-dash-line-the-basis-of-chinas-claim-to-soverei
gnty-over-south-china-sea/469403/

57. Consider the following statements regarding Red Sea


1. It lies between Africa and Asia.
2. Four Asian countries border the Red Sea on the east side
3. The Tropic of Cancer passes through Red Sea

Which of the statements given above is/are correct?


A. 2 only
B. 1 and 3 only
C. 3 only
D. 1, 2 and 3

www.insightsactivelearn.com 55
Total Marks : 200
Test-30(Subject)
( INSTA Prelims Test Series 2021 )

Correct Answer : B

Answer Justification :

Red Sea runs between the two continents Africa and Asia. It is a seawater inlet of the
Indian Ocean. Four African countries border the Red Sea on the western side, while
two Asian countries in east.

The countries that border Red Sea are:

Saudi Arabia

Yemen

Egypt

Sudan

Eritrea

Djibouti

The salinity of the Red Sea is amongst the highest of all oceans in the world at 3.6 to 3.8%.
You might need to adjust your weights accordingly. The Red Sea is home to over 1200 species
of fish and 250 species of coral. Of these, 17% of the fish species and 8% of the coral species
are endemic.

40% of the Red Sea is shallower than 100 meters / 330 feet. And 25% of the Red Sea is less
than 50 meters / 164 feet deep. That’s a lot of area within recreational limits!. The Red Sea is
the northernmost tropical sea in the world, because its waters cross the Tropic of
Cancer.

www.insightsactivelearn.com 56
Total Marks : 200
Test-30(Subject)
( INSTA Prelims Test Series 2021 )

https://www.bbc.com/news/business-56559073

58. Consider the following statements regarding Galilee Basin


1. It is located in Queensland, Australia
2. It is largely known for diamond and rare earth minerals

Which of the statements given above is/are correct?


A. 1 only
B. 2 only
C. Both 1 and 2
D. Neither 1 nor 2

Correct Answer : A

Answer Justification :

The Galilee Basin is spread across 247,000 sq km and is a thermal coal basin. The
basin is located in the Central Australian state of Queensland. Galilee Basin is one of
the largest untapped reserves of coal in the world. Behind Western China, the Galilee
Basin forms the second biggest fossil fuel expansion on the planet, helped with several
proposed ‘mega mines’.

www.insightsactivelearn.com 57
Total Marks : 200
Test-30(Subject)
( INSTA Prelims Test Series 2021 )

Adani has set a 60-year life for the project and aims to support its operations in the country
and potentially help developing Adani’s other mines proposed in the Galilee Basin.

https://indianexpress.com/article/what-is/all-you-need-to-know-about-adanis-mega-mine-coal-pr
oject-in-australia-4882022/

59. Consider the following statements regarding Coronal Mass Ejections (CMEs)
1. It is a giant cloud of solar plasma drenched with magnetic field lines that is blown away from
the Sun often during strong, long-duration solar flares and filament eruptions.
2. It causes disruption of space weather and satellite failures, and power outages, etc

Which of the statements given above is/are correct?


A. 1 only
B. 2 only
C. Both 1 and 2
D. Neither 1 nor 2

Correct Answer : C

Answer Justification :

Both the statements are correct.

Coronal Mass Ejections (CMEs) is a giant cloud of solar plasma drenched with magnetic
field lines that is blown away from the Sun often during strong, long-duration solar flares and
filament eruptions.

A CME contains particle radiation (mostly protons and electrons) and powerful magnetic
fields. They cause disruption of space weather and satellite failures, and power outages, etc

https://indianexpress.com/article/explained/what-are-sunspots-and-why-they-occasionally-spark
-worry-6548800/

60. Consider the following statements regarding Event Horizon Telescope (EHT)
1. EHT project was devised by NASA in 2012 to directly observe the immediate environment of a
black hole.
2. It is a network of 10 radio telescopes on four continents that collectively operate like a single
instrument nearly the size of the Earth.

Which of the statements given above is/are correct?


A. 1 only
B. 2 only
C. Both 1 and 2

www.insightsactivelearn.com 58
Total Marks : 200
Test-30(Subject)
( INSTA Prelims Test Series 2021 )

D. Neither 1 nor 2

Correct Answer : B

Answer Justification :

The EHT is a project of network of 10 radio telescopes on four continents that


collectively operate like a single instrument nearly the size of the Earth.

The EHT project, an international partnership formed in 2012 to directly observe the
immediate environment of a black hole.

Its aim was to capture the first image of a black hole by creating a virtual Earth-sized
telescope.

It is a project to create a large telescope array consisting of a global network of radio


telescopes and combining data from several verylong-baseline interferometry (VLBI) stations
around the Earth

This technique of linking radio dishes across the globe to create an Earth-sized interferometer
has been used to measure the size of the emission regions of the two supermassive black
holes.

https://eventhorizontelescope.org/about

61. Consider the following statements regarding Dragonfly Mission


1. Dragonfly aims to search for signs of microbial alien life on Mars
2. Dragonfly mission is a part of NASA’s New Frontiers program

Which of the statements given above is/are correct?


A. 1 only
B. 2 only
C. Both 1 and 2
D. Neither 1 nor 2

Correct Answer : B

Answer Justification :

Dragonfly aims to search for signs of microbial alien life on Saturn’s moon Titan, while
navigating its earth-like gravity and aerodynamics in the process. Hence Statement 1 is
incorrect.

The mission will succeed NASA’s Cassini probe, which ended its 13-year mission orbiting

www.insightsactivelearn.com 59
Total Marks : 200
Test-30(Subject)
( INSTA Prelims Test Series 2021 )

Saturn in September 2017 by diving into Saturn’s atmosphere.

Statement 2 is correct: Dragonfly mission is a part of NASA’s New Frontiers program, which
includes a series of space exploration missions, which are being conducted with the purpose of
researching several of the Solar System bodies, including the dwarf planet Pluto.

https://www.insightsonindia.com/2019/07/10/nasa-to-launch-dragonfly/

62. Consider the following statements


1. Sagittarius A* is believed to be a super massive galaxy present in the Universe.
2. The Kuiper belt a circumstellar disc in the outer Solar System, extending from the orbit of
Neptune to approximately 50 AU from the Sun.

Which of the statements given above is/are correct?


A. 1 only
B. 2 only
C. Both 1 and 2
D. Neither 1 nor 2

Correct Answer : B

Answer Justification :

Sagittarius A or Sgr A is a complex radio source at the center of the Milky Way which contains
a supermassive black hole. It is located in the constellation Sagittarius, and is hidden from
view at optical wavelengths by large clouds of cosmic dust in the spiral arms of the Milky Way.
Hence Statement 1 is incorrect.

Kuiper Belt — a donut-shaped region of icy bodies beyond the orbit of Neptune. There may be
millions of these icy objects, collectively referred to as Kuiper Belt objects (KBOs) or trans-
Neptunian objects (TNOs), in this distant region of our solar system. It is extending from the
orbit of Neptune to approximately 50 AU from the Sun.

www.insightsactivelearn.com 60
Total Marks : 200
Test-30(Subject)
( INSTA Prelims Test Series 2021 )

63. Consider the following pairs


List I List II
1. Baitarani River - Odisha
2. Salma Dam - Pakistan
3. Kuki Tribes - Mizoram

Which of the pairs given above is/are matched correctly?


A. 1, 2 and 3
B. 1 only
C. 3 only
D. 1 and 3 only

Correct Answer : D

Answer Justification :

The Baitarani River or River Baitarani is one of six major rivers of Odisha, India.
Venerated in popular epics and legends, the Baitarani River is a source of water for
agricultural irrigation.

Salma Dam is located in Afghanistan. Hence Statement 2 is incorrect.

The Kukis, also known as the Chin.They are one of the hill tribe Zo people. In the Chin State of
Myanmar and as Mizo in the Indian state of Mizoram are a number of related Tibeto-
Burman tribal peoples spread throughout the northeastern states of India.

www.insightsactivelearn.com 61
Total Marks : 200
Test-30(Subject)
( INSTA Prelims Test Series 2021 )

64. Glyphosate, often seen in the news, is related to

A. Multidrug Resistance Tuberculosis


B. Weedicide
C. Bio-chemical agent to remove oil spills
D. None of the above

Correct Answer : B

Answer Justification :

https://www.business-standard.com/article/current-affairs/gujarat-firms-sell-monsanto-s-bt3-se
eds-carcinogenic-herbicide-despite-ban-118031300141_1.html

65. Consider the following statements regarding Narcotics Control Bureau (NCB)
1. It was constituted under Drugs Control Act, 1950.
2. It is the nodal drug law enforcement and intelligence agency of India responsible for fighting
drug trafficking and the abuse of illegal substances.
3. It functions under Union Ministry of Home Affairs (MHA).

Which of the statements given above is/are correct?


A. 2 only
B. 1 and 3 only
C. 2 and 3 only
D. 1, 2 and 3

Correct Answer : C

Answer Justification :

www.insightsactivelearn.com 62
Total Marks : 200
Test-30(Subject)
( INSTA Prelims Test Series 2021 )

Narcotics Control Bureau (NCB)

It was created in March 1986 in terms of Section 4 (3) of the Narcotic Drugs and
Psychotropic Substances Act, 1985. Hence Statement 1 is incorrect.

It is the nodal drug law enforcement and intelligence agency of India responsible for
fighting drug trafficking and the abuse of illegal substances. It works in close
cooperation with Customs and Central Excise, State Police Department, Central Bureau of
Investigation (CBI), Central Economic Intelligence Bureau (CEIB) and other Indian intelligence
and law enforcement agencies both at the national and states level.

It also provides resources and training to personnel of India’s law enforcement agencies
in fighting drug trafficking. It also monitors India’s frontiers to track down points where
smuggling activities take place with foreign traffickers.

It comes under the Union ministry of Home Affairs.

66. Which of the following regulatory measures have been taken in the Urban Co-operative Bank sector?
1. The overall priority sector lending target for Urban Co-operative Banks has been decreased.
2. Urban Cooperative Banks (UCBs) with assets of 50 crore and above were brought under the
CRILC reporting framework.

Which of the statements given above is/are correct?


A. 1 only
B. 2 only
C. Both 1 and 2
D. Neither 1 nor 2

Correct Answer : D

Answer Justification :

None of the statements given above are correct.

To promote financial inclusion, the overall priority sector lending target for Urban
Co-operative Banks has been increased from the present level of 40 per cent of
adjusted net bank credit (ANBC) or credit equivalent amount of off-balance sheet
exposure (CEOBSE), whichever is higher, to 75 per cent of ANBC or CEOBSE, whichever
is higher by March 31, 2024

Urban Cooperative Banks (UCBs) with assets of `500 crore and above were brought
under the CRILC reporting framework. Accordingly, UCBs shall report credit
information, including classification of an account as Special Mention Account
(SMA), on all borrowers having aggregate exposures of `5 crore and above with them to
www.insightsactivelearn.com 63
Total Marks : 200
Test-30(Subject)
( INSTA Prelims Test Series 2021 )

CRILC

67. Which of the following broad parameters is/are used to calculate the Digital Payments Index (DPI)?
1. Payment Enablers
2. Payment Performance
3. Consumer Centricity

Select the correct answer using the code given below:


A. 1 and 2 only
B. 2 and 3 only
C. 1 and 3 only
D. 1, 2 and 3

Correct Answer : D

Answer Justification :

Reserve Bank of India has constructed a composite Digital Payments Index (DPI) to capture
the extent of digitisation of payments across the country.

The RBI-DPI comprises of 5 broad parameters that enable measurement of deepening and
penetration of digital payments in the country over different time periods. These parameters
are: (i) Payment Enablers (weight 25%), (ii) Payment Infrastructure – Demand-side
factors (10%), (iii) Payment Infrastructure – Supply-side factors (15%), (iv) Payment
Performance (45%) and (v) Consumer Centricity (5%).

The RBI-DPI has been constructed with March 2018 as the base period, i.e. DPI score
for March 2018 is set at 100.

68. Consider the following statements regarding the Consumer Price Index for Industrial Workers (CPI-IW):
1. It is compiled and disseminated by the National Statistical Office (NSO).
2. The base year of the CPI-IW series is 2016.
3. Fuel & Light group has highest weightage in the Index.

Which of the statements given above is/are correct?


A. 1 only
B. 2 only
C. 1, 2 and 3
D. None

Correct Answer : B

www.insightsactivelearn.com 64
Total Marks : 200
Test-30(Subject)
( INSTA Prelims Test Series 2021 )

Answer Justification :

The CPI-IW is compiled and disseminated by the Labour Bureau on a monthly basis.

Hence, statement 1 is incorrect.

It measures changes in the retail prices of a fixed basket of goods and services being
consumed by an average working-class family. Apart from serving as a guide for policy
formulations, Prices and Inflation 167 these index numbers are utilized for fixing/revising
wages, regulating the dearness allowances paid to large number of manual workers and
Central/ State Govt. employees.

To capture the latest consumption pattern of working-class family, Labour Bureau


has revised the base year of the existing CPI-IW series 2001=100 to a more recent
base year 2016=100.

Hence, statement 2 is correct.

The new series of CPI-IW covers the industrial workers from the existing seven sectors viz.
Factories, Mines, Plantation, Railways, Public Motor Transport Undertakings, Electricity
Generating & Distributing Establishments and Ports & Docks. The new series has a wider
coverage in terms of sample size, number of centres, markets/outlets, items etc.

Hence, statement 3 is incorrect.

69. Which of following measures could be used by the government to control the sudden spike in specific food
items?

www.insightsactivelearn.com 65
Total Marks : 200
Test-30(Subject)
( INSTA Prelims Test Series 2021 )

1. Temporary ban on the export of inflated item.


2. Imposition of stock limit under the Essential Commodities Act
3. Temporary prohibition on the import of inflated item.

Select the correct answer using the code given below:


A. 1 and 2 only
B. 3 only
C. 2 and 3 only
D. 2 only

Correct Answer : A

Answer Justification :

The Government reviews the price situation regularly and has taken number of measures from
time to time to stabilize prices of food items.

In the wake of rising prices of pulses, onion and potato, the Government has taken several
steps to improve the availability of these commodities and make them available to consumers
at affordable prices.

These include: i. Banning the export of onion w.e.f. 14.09.2020, revoked w.e.f.
1.01.2021.

Hence, statement 1 is correct.

ii. Imposition of stock limit on onion under the EC Act w.e.f. 23.10.2020 to prevent
hoarding, lapsed 31.12.2020.

Hence, statement 2 is correct.

iii. Easing of restrictions on imports, facilitating imports at integrated check-posts,


issuance of licenses for imports and reduction in import duties.

Hence, statement 3 is incorrect.

These measures have resulted in increased imports of onion, tur dal and masur dal in the
country and resultant cooling of prices.

70. Which of the following is/are targets of India’s Intended Nationally Determined Contribution (INDC)?
1. To Reduce the emissions intensity of its GDP by 33 to 35 per cent below 2005 levels by the
year 2030.
2. To achieve 75 per cent of cumulative electric power installed capacity from non-fossil fuel
sources by 2030.
3. To create additional carbon sink equivalent to 25 to 30 billion tons of carbon dioxide by 2030.

www.insightsactivelearn.com 66
Total Marks : 200
Test-30(Subject)
( INSTA Prelims Test Series 2021 )

Select the correct answer using the code given below:


A. 1 and 2 only
B. 1 only
C. 2 and 3 only
D. 1 and 3 only

Correct Answer : B

Answer Justification :

As mandated in the UNFCCC and its Paris Agreement, the climate actions of the
developing countries would have to be supported by finance flows from the developed to the
developing countries.

The Nationally Determined Contribution (NDC) submitted by the country has been formulated
keeping in mind the developmental imperatives of the country and is on a “best effort basis”.

In its NDC, India has sought to reduce the emissions intensity of its GDP by 33 to 35
per cent below 2005 levels by the year 2030;

Hence, statement 1 is correct.

Achieve 40 per cent of cumulative electric power installed capacity from non-fossil
fuel sources by 2030;

Hence, statement 2 is incorrect.

Enhance forest and tree cover to create additional carbon sink equivalent to 2.5 to 3
billion tons of carbon dioxide by 2030.

Hence, statement 3 is incorrect.

71. Consider the following statements regarding the National Adaptation Fund on Climate Change (NAFCC):
1. It is a Central Sector Scheme which was announced in the budget 2021-22.
2. The objective of the fund is to meet the cost of adaptation to climate change for the State and
Union Territories of India that are particularly vulnerable to the adverse effects of climate
change.
3. NABARD is the National Implementing Entity (NIE) for Adaptation Fund (AF) under Kyoto
Protocol.

Which of the statements given above is/are correct?


A. 1 only
B. 2 and 3 only
C. 1, 2 and 3
D. None

www.insightsactivelearn.com 67
Total Marks : 200
Test-30(Subject)
( INSTA Prelims Test Series 2021 )

Correct Answer : B

Answer Justification :

The National Adaptation Fund for Climate Change (NAFCC) is a Central Sector
Scheme which was set up in the year 2015-16.

Hence, statement 1 is incorrect.

The National Adaptation Fund for Climate Change (NAFCC) was established in
August, 2015 to meet the cost of adaptation to climate change for the State and
Union Territories of India that are particularly vulnerable to the adverse effects of
climate change. Government has set up a budget provision of Rs.350 crores for the
year 2015-16 and 2016-17, with an estimated requirement of Rs. 181.5 crores for
financial year 2017-18 for NAFCC.

Hence, statement 2 is correct.

The projects under NAFCC prioritizes the needs that builds climate resilience in the areas
identified under the SAPCC (State Action Plan on Climate Change) and the relevant Missions
under NAPCC (National Action Plan on Climate Change).

72. Consider the following pairs of important session of the Conference of Parties (COP)
of the United Nations Framework Convention on Climate Change (UNFCCC) and
key decisions taken:
COP Sessions: Key Decisions:
1. COP-15: Copenhagen Formalized the setting up of the GCF
2. COP-16: Cancun First quantification of climate finance
3. COP-21: Paris Concept of Nationally Determined Contribution

Which of the pairs given above is/are correctly matched?


A. 1 and 2 only
B. 3 only
C. 2 only
D. 1 and 3 only

Correct Answer : B

Answer Justification :

www.insightsactivelearn.com 68
Total Marks : 200
Test-30(Subject)
( INSTA Prelims Test Series 2021 )

73. Consider the following statements:

1. India has the largest green bond market among the emerging markets.
2. India is one of the founding members of the European Commission-led International Platform
on Sustainable Finance (IPSF).

Which of the statements given above is/are correct?


A. 1 only
B. 2 only
C. Both 1 and 2
D. Neither 1 nor 2

Correct Answer : B

Answer Justification :

India has the second largest green bond market among the emerging markets after
China.

Hence, statement 1 is incorrect.

www.insightsactivelearn.com 69
Total Marks : 200
Test-30(Subject)
( INSTA Prelims Test Series 2021 )

With the objective of promoting global cooperation in sustainable finance, India


joined the European Commission-led International Platform on Sustainable Finance
(IPSF) in October 2019 as one of the founding members.

Hence, statement 2 is correct.

The 16-member jurisdiction platform accounts for 50 per cent of the world
population, almost 50 per cent of global GDP and about 55 per cent of global GHG
emissions. The platform is designed as a member driven informal and inclusive entity.

The Platform acknowledges that the global nature of financial markets has the great potential
to help finance the transition to a green, low-carbon and climate resilient economy by linking
financing needs to global sources of funding. The platform would act as a forum for facilitating
exchanges and, where appropriate, coordinating efforts on initiatives and approaches to
environmentally sustainable finance, in particular in the areas of taxonomies, disclosures,
standards and labels, while acknowledging differences in national and regional contexts.

74. The World Solar Bank has been proposed to be launched by:

A. World Bank (WB)


B. International Solar Alliance (ISA)
C. World Energy Council
D. The International Energy Agency (IEA)

Correct Answer : B

Answer Justification :

International Solar Alliance (ISA) has recently launched two new initiatives – a ‘World
Solar Bank’ and ‘One Sun One World One Grid Initiative’ - of global import that are
poised to be instrumental in bringing about solar energy revolution globally.

Hence, option (b) is correct.

www.insightsactivelearn.com 70
Total Marks : 200
Test-30(Subject)
( INSTA Prelims Test Series 2021 )

The proposed World Solar Bank would cater to the need for dedicated financing window for
solar energy projects across the members of ISA. It is expected to provide low-cost
financing at favorable terms for solar energy projects as well as engage in co-
financing with other multilateral/bilateral development financial institutions.

The ‘One Sun One World One Grid’ vision was laid down by the Hon’ble Prime Minister of
India at the first assembly of the ISA. The initiative aims to create an interconnected green
grid that will enable solar energy generation in regions with high potential and facilitate its
evacuation to demand centers

75. The Chabahar Port, sometimes seen in news, is located in

A. Iran
B. Afghanistan
C. Iraq
D. Oman

Correct Answer : A

Answer Justification :

Chabahar Port is a seaport in Chabahar located in southeastern Iran, on the Gulf of Oman. It
serves as Iran's only oceanic port, and consists of two separate ports named Shahid Kalantari
and Shahid Beheshti, each of which has five berths.

www.insightsactivelearn.com 71
Total Marks : 200
Test-30(Subject)
( INSTA Prelims Test Series 2021 )

76. Consider the following statements regarding Langrangian point


1. It is a point where the attraction by the Sun and the Earth becomes equal.
2. Langrangian point experience the highest gravitational force.

Which of the statements given above is/are correct?


A. 1 only
B. 2 only
C. Both 1 and 2
D. Neither 1 nor 2

Correct Answer : A

Answer Justification :

Lagrangian points are the locations in space where the combined gravitational pull of
two large masses roughly balance each other. Any small mass placed at that location will
remain at constant distances relative to the large masses. Hence Statement 2 is incorrect.

There are five such points in Sun-Earth system and they are denoted as L1, L2, L3, L4 and L5.
A halo orbit is a periodic three-dimensional orbit near the L1, L2 or L3.

It is a point where the attraction by the Sun and the Earth becomes equal. The point
doesn’t experience gravitational force.

77. Consider the following statements regarding LIGO- India project

www.insightsactivelearn.com 72
Total Marks : 200
Test-30(Subject)
( INSTA Prelims Test Series 2021 )

1. It is piloted by Department of Atomic Energy (DAE) and Department of Science and


Technology (DST).
2. It is a massive observatory for detecting cosmic gravitational waves and for carrying out
experiments.

Which of the statements given above is/are correct?


A. 1 only
B. 2 only
C. Both 1 and 2
D. Neither 1 nor 2

Correct Answer : C

Answer Justification :

Both the statements are correct.

LIGO is a massive observatory for detecting cosmic gravitational waves and for carrying
out experiments.

The objective is to use gravitational-wave observations in astronomical studies.

The project operates three gravitational-wave (GW) detectors. Two are at Hanford,
Washington, north-western US, and one is at Livingston in Louisiana, south-eastern US.

The proposed LIGO India project aims to move one advanced LIGO detector from Hanford to
India.

About LIGO- India project:

It is piloted by Department of Atomic Energy (DAE) and Department of Science and


Technology (DST).

The LIGO-India project will be jointly coordinated and executed by three Indian research
institutions: The Inter-University Centre for Astronomy and Astrophysics (IUCAA), Pune and
Department of Atomic Energy organizations: Institute for Plasma Research (IPR), Gandhinagar
and the Raja Ramanna Centre for Advanced Technology (RRCAT), Indore.

Benefits for India:

1. The project will bring unprecedented opportunities for scientists and engineers to dig
deeper into the realm of gravitational wave and take global leadership in this new
astronomical frontier.

2. The LIGO-India project will also bring considerable opportunities in cutting-edge technology
for the Indian industry which will be engaged in the construction of the eight-km long beam
tube at ultra-high vacuum on a levelled terrain.

www.insightsactivelearn.com 73
Total Marks : 200
Test-30(Subject)
( INSTA Prelims Test Series 2021 )

3. With its establishment, India will join the global network of gravitational wave detectors.

4. Establishing an observatory in India also assumes importance because the further the
distance between the observatories, the greater will be the accuracy in locating gravity waves.

78. Which of the following is/are the applications of Helium?


1. Semiconductors and Optics
2. Leak Detection
3. Space Launch Vehicles

Select the correct answer using the code given below


A. 2 and 3 only
B. 2 only
C. 1 and 3 only
D. 1, 2 and 3

Correct Answer : D

Answer Justification :

Helium is a chemical element with the symbol He and atomic number 2. It is a colorless,
odorless, tasteless, non-toxic, inert, monatomic gas, the first in the noble gas group in the
periodic table.

Its boiling point is the lowest among all the elements. Helium is the second lightest and second
most abundant element in the observable universe (hydrogen is the lightest and most
abundant).

It is present at about 24% of the total elemental mass, which is more than 12 times the mass of
all the heavier elements combined.

www.insightsactivelearn.com 74
Total Marks : 200
Test-30(Subject)
( INSTA Prelims Test Series 2021 )

79. Consider the following statements regarding Quark


1. It is a subatomic particle
2. Charm quarks are much lighter than the up and down quarks

Which of the statements given above is/are correct?


A. 1 only
B. 2 only
C. Both 1 and 2
D. Neither 1 nor 2

Correct Answer : A

Answer Justification :

A quark is a type of elementary particle and a fundamental constituent of matter.


Quarks combine to form composite particles called hadrons, the most stable of which are
protons and neutrons, the components of atomic nuclei.

Quarks have various intrinsic properties, including electric charge, mass, color charge, and
spin. They are the only elementary particles in the Standard Model of particle physics to
experience all four fundamental interactions, also known as fundamental forces
(electromagnetism, gravitation, strong interaction, and weak interaction), as well as the only
known particles whose electric charges are not integer multiples of the elementary charge.

There are six types, known as flavors, of quarks: up, down, charm, strange, top, and

www.insightsactivelearn.com 75
Total Marks : 200
Test-30(Subject)
( INSTA Prelims Test Series 2021 )

bottom. Up and down quarks have the lowest masses of all quarks. Hence Statement 2
is incorrect.

80. Consider the following statements regarding National Population Register


1. It includes both Indian citizens as well as a foreign citizens
2. It will be conducted by Ministry of Statistics and Programme Implementation
3. It is mandatory for every usual resident of India to register in the NPR.

Which of the statements given above is/are correct?


A. 1 and 2 only
B. 2 only
C. 1 and 3 only
D. 1, 2 and 3

Correct Answer : C

Answer Justification :

National Population Register (NPR) will be conducted by the Office of the Registrar
General of India (RGI) under the Home Ministry. It is a Register of usual residents of the
country. Hence Statement 2 is incorrect.

It is being prepared at the local (Village/sub-Town), sub-District, District, State and National
level under provisions of the Citizenship Act 1955 and the Citizenship (Registration of Citizens
and issue of National Identity Cards) Rules, 2003. It is mandatory for every usual resident
of India to register in the NPR.

Objective: To create a comprehensive identity database of every usual resident in the country.

It includes both Indian citizens as well as a foreign citizens.

81. Who appoints Chief Justice of India?

A. Union Cabinet
B. Parliament
C. Collegium in Supreme Court
D. President of India

Correct Answer : D

Answer Justification :

www.insightsactivelearn.com 76
Total Marks : 200
Test-30(Subject)
( INSTA Prelims Test Series 2021 )

The Constitution of India does not mention any procedure for appointing the CJI. Article 124
(1) of the Constitution merely says, “there shall be a Supreme Court of India consisting of a
Chief Justice of India.”

Clause (2) of Article 124 of the Constitution says that every Judge of the Supreme Court shall
be appointed by the President. Thus, in the absence of a constitutional provision, the
procedure to appoint CJI relies on convention.

The outgoing CJI recommends his successor – a practice, which is strictly based on seniority.
The Union Law Minister forwards the recommendation to the Prime Minister who, in turn,
advises the President.

Thus, after a CJI retires at the age of 65, the senior most judge in the Supreme Court becomes
the CJI. Seniority, however, is not defined by age, but by the number of years a judge has been
serving in the top court of the country.

In cases where the two judges have the same seniority, other factors, like who among the two
has more years of experience in the High Court or whether any of them was nominated from
the bar directly, or who took the oath first, come into play.

82. Consider the following statements regarding Investor Education and Protection Fund
1. It has been established under provision of the Companies Act, 2013.
2. The amounts such as dividends, applications money, matured deposits etc, which have
remained unpaid or unclaimed for a period of 7 years are required to be transferred to the
IEPF.

Which of the statements given above is/are correct?


A. 1 only
B. 2 only
C. Both 1 and 2
D. Neither 1 nor 2

Correct Answer : C

Answer Justification :

Both the statements are correct.

Investor Education and Protection Fund has been established under provision of the
Companies Act, 2013. The amounts such as dividends, applications money, matured deposits
etc, which have remained unpaid or unclaimed for a period of 7 years are required to be
transferred to the IEPF.

The Amounts credited to IEPF are maintained under the Consolidated Fund of India (Article
266 of the Constitution. It helps to promote investor awareness and protection of investor

www.insightsactivelearn.com 77
Total Marks : 200
Test-30(Subject)
( INSTA Prelims Test Series 2021 )

interests

http://www.iepf.gov.in/

83. Consider the following statements regarding Lithium Sulphur (Li-S) Battery
1. Li-S batteries are generally considered to be the successors of the Lithium-ion (Li-ion)
batteries.
2. The cost of production of Li-S battery is way costlier than normal batteries.

Which of the statements given above is/are correct?


A. 1 only
B. 2 only
C. Both 1 and 2
D. Neither 1 nor 2

Correct Answer : A

Answer Justification :

Lithium sulphur batteries are generally considered to be the successors of the


Lithium ion (Li-ion) batteries because of their lower cost of production, energy
efficiency and improved safety. Their cost of production is lower because sulphur is
abundantly available.

There have been some difficulties when it comes to commercialising these batteries, mainly
due to their short life cycle and poor instantaneous power capabilities.

The Li–S batteries are promising because of the high energy density, low cost, and natural
abundance of sulfur material. However, these advantages can be achieved only when the Li–S
battery uses elemental sulfur as the cathode active material and the sulfur approaches the
theoretical capacity with low process cost.

In recent years, great improvement in the cycling performances of Li–S batteries has been
made; however, all these achievements are obtained in exchange for the energy density and
process cost.

84. Consider the following statements regarding Biosimilar medicine


1. It is a biologic medical product that is almost an identical copy of an original product that is
manufactured by a different company.
2. Biosimilars involve developing equivalent of a chemical entity-the Active Pharmaceutical
Ingredient.
3. They are officially approved versions of original “innovator” products and can be
manufactured when the original product’s patent expires.

www.insightsactivelearn.com 78
Total Marks : 200
Test-30(Subject)
( INSTA Prelims Test Series 2021 )

Which of the statements given above is/are correct?


A. 1 and 2 only
B. 1 and 3 only
C. 2 and 3 only
D. 1, 2 and 3

Correct Answer : B

Answer Justification :

Biosimilar medicine

It is a biologic medical product that is almost an identical copy of an original product


that is manufactured by a different company. They are officially approved versions of
original “innovator” products and can be manufactured when the original product’s
patent expires.

Characteristics:

• Biological medicines contain active substances from a biological source, such as living
cells or organisms.

• Most biological medicines in current clinical use contain active substances made of
proteins.

Difference between biosimilars and generics:

• Biosimilars involve developing equivalent of biological entity while generics involve


developing equivalent of a chemical entity-the Active Pharmaceutical Ingredient.

85. Consider the following statements regarding Microbial Fuel Cells


1. It is a device that converts chemical energy to electrical energy by the action of
microorganisms.
2. Microbial fuel cells have been installed at National Zoological Park, Delhi.

Which of the statements given above is/are correct?


A. 1 only
B. 2 only
C. Both 1 and 2
D. Neither 1 nor 2

www.insightsactivelearn.com 79
Total Marks : 200
Test-30(Subject)
( INSTA Prelims Test Series 2021 )

Correct Answer : A

Answer Justification :

Microbial Fuel Cells

Microbial fuel cells have been installed at a zoo in London.

Using these cells, a plant has taken the botanical world's first selfie.

What are microbial fuel cells?

A device that converts chemical energy to electrical energy by the action of microorganisms.

How it works?

Under sunlight, plants produce sugars and oxygen from water and CO2 (photosynthesis).
These sugars do not remain in the leaves, but are transported throughout the plant to the stem
and roots. Some of these sugars are excreted by the roots as a waste product from the plant.

Soil micro-organisms break this down further, releasing energy. This energy is captured using
an anode (minus) and a cathode (plus) and charge a super capacitor.

When the super capacitor is full, the power is discharged and a photo is taken. It is a bio-
electrochemical system that uses bacteria as the catalyst to oxidize organic and inorganic
matter, and consequently, generate electric current out of it.

86. NetWire, sometime seen in the news, is largely related to

A. Cyber Attack
B. Hardware Essentials
C. Last mile connectivity
D. None of the above

Correct Answer : A

Answer Justification :

NetWire is a remote access Trojan focused on password stealing and keylogging, as


well as including remote control capabilities. This threat has been used by malicious
groups since 2012 and distributed through various social engineering campaigns (malspam).
Recently, NetWire has been distributed as a second payload using Microsoft Word documents
via GuLoader phishing waves.

The NetWire RAT is malicious software that emerged in the wild in 2012. This multi-platform
malware has since undergone several upgrades and was identified in different types of attacks

www.insightsactivelearn.com 80
Total Marks : 200
Test-30(Subject)
( INSTA Prelims Test Series 2021 )

that range from Nigerian scammers to advanced persistent threat (APT) attacks.

87. Which of the following is/are the advantages of Nano Photonics?


1. Lower Noise
2. Less dissipation of energy
3. Lower input signal

Select the correct answer using the code given below


A. 2 only
B. 1 and 3 only
C. 1 and 2 only
D. 1, 2 and 3

Correct Answer : D

Answer Justification :

88. Which of the following is/are included in First Generation Bio-Fuels?


1. Bioalcohol
2. Biodiesel
3. Biohydrogen

Select the correct answer using the code given below


A. 1 and 3 only
B. 1 and 2 only
C. 1, 2 and 3

www.insightsactivelearn.com 81
Total Marks : 200
Test-30(Subject)
( INSTA Prelims Test Series 2021 )

D. None

Correct Answer : B

Answer Justification :

Biofuels are liquid or gaseous fuels primarily produced from biomass, and can be used to
replace or can be used in addition to diesel, petrol or other fossil fuels for transport,
stationary, portable and other applications. Crops used to make biofuels are generally either
high in sugar (such as sugarcane, sugarbeet, and sweet sorghum), starch (such as maize and
tapioca) or oils (such as soybean, rapeseed, coconut, sunflower).

Categories of biofuels

Biofuels are generally classified into three categories. They are

1. First generation biofuels - First-generation biofuels are made from sugar,


starch, vegetable oil, or animal fats using conventional technology. Common
first-generation biofuels include Bioalcohols, Biodiesel, Vegetable oil,
Bioethers, Biogas.

https://vikaspedia.in/energy/energy-production/bio-energy/biofuels

89. Consider the following statements regarding Krishi Vigyan Kendras


1. It was setup based on the recommendation of Education Commission (1964-66).
2. They are innovative institutions for imparting vocational training to the practicing farmers,
school dropouts and field level extension.
3. The first KVK, on a pilot basis, was established at Rajasthan.

Which of the statements given above is/are correct?


A. 1 and 2 only
B. 2 only
C. 1 and 3 only
D. 1, 2 and 3

Correct Answer : A

Answer Justification :

ICAR mooted the idea of establishing Krishi Vigyan Kendras (Agricultural Science
Centres) as innovative institutions for imparting vocational training to the practicing
farmers, school dropouts and field level extension functionaries.

www.insightsactivelearn.com 82
Total Marks : 200
Test-30(Subject)
( INSTA Prelims Test Series 2021 )

The ICAR Standing Committee on Agricultural Education, in its meeting held in August, 1973,
observed that since the establishment of Krishi Vigyan Kendras (KVKs) was of national
importance which would help in accelerating the agricultural production as also in improving
the socio-economic conditions of the farming community, the assistance of all related
institutions should be taken in implementing this scheme. The ICAR, therefore, constituted a
committee in 1973 headed by Dr. Mohan Singh Mehta of Seva Mandir, Udaipur (Rajasthan),
for working out a detailed plan for implementing this scheme. The Committee submitted its
report in 1974.

The first KVK, on a pilot basis, was established in 1974 at Puducherry (Pondicherry)
under the administrative control of the Tamil Nadu Agricultural University,
Coimbatore. Hence Statement 3 is incorrect.

At present there are 713 KVKs, out of which 498 are under State Agricultural Universities
(SAU) and Central Agricultural University (CAU), 63 under ICAR Institutes, 101 under NGOs,
38 under State Governments, and the remaining under other educational institutions.

90. Consider the following statements regarding the Agriculture Infrastructure Fund:
1. It provides for medium to long term debt financing facility for investment in viable projects for
post-harvest management infrastructure and community farming assets.
2. All loans under this financing facility will have interest subvention of 3 per cent per annum up
to a limit of 20 lakhs.
3. Credit guarantee coverage will be available for eligible borrowers from this financing facility
up to certain specified limit.

Which of the statements given above is/are correct?


A. 1 and 2 only
B. 2 and 3 only
C. 3 only
D. 1 and 3 only

Correct Answer : D

Answer Justification :

www.insightsactivelearn.com 83
Total Marks : 200
Test-30(Subject)
( INSTA Prelims Test Series 2021 )

91. Which among the following agriculture commodities has the highest share in total agricultural export value in
last five years?

A. Marine Products
B. Spices
C. Sugar
D. Tea

Correct Answer : A

Answer Justification :

An analysis of last six years of the share of top ten agricultural commodities in total value of
agricultural export shows that there have been significant changes in the composition of
agriexports. The share of marine products in total agricultural export value has
remained the largest over the period.

www.insightsactivelearn.com 84
Total Marks : 200
Test-30(Subject)
( INSTA Prelims Test Series 2021 )

Hence, option (a) is correct.

92. Animal Husbandry Infrastructure Development Fund (AHIDF) can be used to establish:
1. Dairy processing and Value addition infrastructure
2. Meat processing and Value addition infrastructure
3. Animal feed plant

Select the correct answer using the code given below:


A. 1 and 2 only
B. 2 and 3 only
C. 1 and 3 only
D. 1, 2 and 3

Correct Answer : D

Answer Justification :

As a part of the Atma Nirbhar Bharat Abhiyan stimulus package, a ` 15000 crores Animal
Husbandry Infrastructure Development Fund (AHIDF) has been set up.

The AHIDF will incentivize investments by individual entrepreneurs, private


companies including MSME, farmers producers organizations(FPOs) and Section 8
companies to establish (i) dairy processing and value addition infrastructure (ii) meat
processing and value addition infrastructure, and (iii) animal feed plant.

www.insightsactivelearn.com 85
Total Marks : 200
Test-30(Subject)
( INSTA Prelims Test Series 2021 )

93. Consider the following statements regarding the Operation Greens scheme:
1. It is a central sector scheme which is being implemented by the ministry of Agriculture and
Farmer Welfare.
2. This scheme is meant for intervention in the market during price rise.

Which of the statements given above is/are correct?


A. 1 only
B. 2 only
C. Both 1 and 2
D. Neither 1 nor 2

Correct Answer : D

Answer Justification :

None of the statements given above are correct.

MoFPI is implementing a central sector scheme “Operation Greens – A scheme for


integrated development of Tomato, Onion and Potato (TOP) value chain” to provide
support to farmers when prices of agri produce is low.

This scheme is not meant for intervention in the market during price rise.

Under the short term- price stabilization measures of the scheme, there is a provision for 50
per cent subsidy on cost of transportation and storage for evacuation of surplus production
from producing area to the consumption center during the glut situation

94. Which of the following sectors is/are part of eight core industries?
1. Natural Gas
2. Cement
3. Textiles
4. Steel

Select the correct answer using the code given below:


A. 1, 2 and 3 only
B. 2, 3 and 4 only
C. 1, 2 and 4 only
D. 1 and 4 only

Correct Answer : C

Answer Justification :

www.insightsactivelearn.com 86
Total Marks : 200
Test-30(Subject)
( INSTA Prelims Test Series 2021 )

The eight-core industries that support infrastructure, such as coal, crude oil, natural
gas, refinery products, fertilizers, steel, cement, and electricity have a total weight of
nearly 40 percent in the IIP.

Textile sector is not part of eight core industries.

95. Consider the following statements:


1. India is the largest producer of crude steel.
2. India imports more coal than its domestic production.

Which of the statements given above is/are correct?


A. 1 only
B. 2 only
C. Both 1 and 2
D. Neither 1 nor 2

Correct Answer : D

Answer Justification :

None of the statements given above are correct.

India is the second-largest producer of crude steel only after China. India is also the
second largest consumer of steel.

However, its per capita total finished steel consumption was around 74.7 kg during FY20 as
against the global average of 229 kg. Further, the capacity utilization in crude steel plants
continues to be low.

Coal is the one of the most important and abundant fossil fuel in India. It accounts
for 55 per cent of the country’s energy needs. Coal is not only the primary source of
energy in the country but is also used as an intermediary by many industries such as steel,
sponge iron, cement, paper, brick-kilns, etc.

In the FY20, the production of raw coal in India was 729.1 million tonnes (MnT) with
a minuscule growth of 0.05 per cent over the previous year. India is also an importer
of coal importing 248.54 MT of coal in FY20, a growth of 5.7 per cent over FY19.

www.insightsactivelearn.com 87
Total Marks : 200
Test-30(Subject)
( INSTA Prelims Test Series 2021 )

96. Which of the following organizations has published the World Investment Report 2020?

A. World Trade Organization (WTO)


B. World Bank
C. United Nations Conference on Trade and Development (UNCTAD)
D. World Economic Forum (WEF)

Correct Answer : C

Answer Justification :

India improved its position from 12th in 2018 to 9th in 2019 in the list of the world’s
largest FDI recipients according to the latest World Investment Report 2020 by
United Nations Conference on Trade and Development (UNCTAD).

97. Consider the following statements regarding difference between CPI and GDP deflator
1. CPI includes the prices of imported goods, but they are not included in GDP deflator.
2. The weights are constant in CPI, but they differ according to production level of each good in
GDP deflator.

Which of the statement above is/are correct?


A. 1 only
B. 2 only
C. Both 1 and 2

www.insightsactivelearn.com 88
Total Marks : 200
Test-30(Subject)
( INSTA Prelims Test Series 2021 )

D. Neither 1 nor 2

Correct Answer : C

Answer Justification :

Both the statements given above are correct.

Consumer Price Index or CPI as it is commonly called is an index measuring retail inflation
in the economy by collecting the change in prices of most common goods and services. The
CPI is calculated with reference to a base year, which is used as a benchmark. The CPI is
used to calculate the inflation levels in an economy. This can be further used to compute
the cost of living. This also provides insights as to how much a consumer can spend to be on
par with the price change. (Study different types of CPI, their releasing authority, base
year and sectorial weightage)

The Gross Domestic Product (GDP) deflator: The Gross Domestic Product (GDP)
deflator is a measure of general price inflation. It is calculated by dividing nominal GDP by
real GDP and then multiplying by 100. Ministry of Statistics and Programme
Implementation (MOSPI) comes out with GDP deflator in National Accounts Statistics as
price indices. The base of the GDP deflator is revised when base of GDP series is changed.
Present Base year is 2011.

CPI may differ from GDP deflator because

1. The goods purchased by consumers do not represent all the goods which are
produced in a country. GDP deflator takes into account all such goods and services.

2. CPI includes prices of goods consumed by the representative consumer; hence it includes
prices of imported goods. GDP deflator does not include prices of imported goods.

3. The weights are constant in CPI – but they differ according to production level of
each good in GDP deflator.

98. Which of the following is/are important characteristics of recession?


1. Industries resort to ‘price cuts’ to sustain their business
2. Inflation remains lower
3. Fall in the employment rate

Select the correct answer using the code given below:


A. 1 and 2 only
B. 2 and 3 only
C. 1 and 3 only
D. 1, 2 and 3

www.insightsactivelearn.com 89
Total Marks : 200
Test-30(Subject)
( INSTA Prelims Test Series 2021 )

Correct Answer : D

Answer Justification :

All the statements given above are correct.

Recession: Recession is a part of the business cycle accompanied by a general decline in


economic activity. It is characterised by reduction demand, consumption, investment
and increasing unemployment rates etc

Major traits of recession may be summed up as follows:

(i) there is a general fall in demand as economic activities takes a downturn;

(ii) inflation remains lower or/and shows further signs of falling down;

(iii) employment rate falls/unemployment rate grows;

(iv) Industries resort to ‘price cuts’ to sustain their business.

99. Consider the following is/are objectives of Competition Commission of India (CCI):
1. To prevent practices having adverse effect on competition.
2. To promote and sustain competition in markets.
3. To protect the interests of consumers

Select the correct answer using the code given below:


A. 2 and 3 only
B. 3 only
C. 1 and 3 only
D. 1, 2 and 3

Correct Answer : D

Answer Justification :

All the statements given above are correct.

Competition Commission of India is a statutory body of the Government of India,


responsible for enforcing the Competition Act, 2002 throughout India and to prevent activities
that have an adverse effect on competition.

The Commission consists of one Chairperson and six Members as per the Competition
Act who shall be appointed by the Central Government.

It is the duty of the Commission to eliminate practices having adverse effect on competition,

www.insightsactivelearn.com 90
Total Marks : 200
Test-30(Subject)
( INSTA Prelims Test Series 2021 )

promote and sustain competition, protect the interests of consumers and ensure freedom of
trade in the markets of India. The Commission is also required to give opinion on competition
issues on a reference received from a statutory authority established under any law and to
undertake competition advocacy, create public awareness and impart training on competition
issues.

Objectives of the Commission:

To promote and sustain competition in markets.


To protect the interests of consumers.
To ensure freedom of trade.

100. Which of the following organization has developed the ‘Harmonized System Nomenclature.’ (HSN) Code:

A. World Trade Organization (WTO)


B. World Customs Organization (WCO)
C. World Economic Forum (WEF)
D. World Intellectual Property Organization (WIPO)

Correct Answer : B

Answer Justification :

HSN stands for ‘Harmonized System Nomenclature.’ The WCO (World Customs
Organization) developed it as a multipurpose international product nomenclature that first
came into effect in 1988 with the vision of facilitating the classification of goods all over the
World in a systematic manner.

What does the HS code mean? - Harmonised System, or simply ‘HS’: Developed by the
World Customs Organization (WCO).

1. Called the “universal economic language” for goods.


2. It is a multipurpose international product nomenclature.
3. The system currently comprises of around 5,000 commodity groups.
4. HS Code is also known as HSN Code in India. Goods are classified into
Harmonized System of Nomenclature or HSN. It is used up to 8-digit level.
5. HSN classification is widely used for taxation purposes by helping to identify the
rate of tax applicable to a specific product in a country that is under review. It can
also be used in calculations that involve claiming benefits.

www.insightsactivelearn.com 91

You might also like